You are on page 1of 131

CMA Ontario

Accelerated Program

MANAGEMENT ACCOUNTING
MODULE 3

CMA Ontario – January 2008


Module 3 - Management Accounting

(this page left blank)

Page 2 CMA Ontario – January 2008


Module 3 - Management Accounting

Table of Contents

Textbook Ch* Page

1. Role of the Management Accountant 1 5

2. Cost Classifications 2 9

3. Job Order Costing 4 17

4. Process Costing and Spoilage 17, 18 26

5. Cost Behaviour 10 37

6. Activity Based Costing 5 44

7. Cost Allocations – Service Department Allocations 14 52

8. Joint and By-Product Costing 15 63

9. In-Class Problems 71

10. Multiple Choice Questions 87

11. Powerpoint Slides 105

* note that this module comprises mostly of solutions to selected problems in the
textbook.

Page 3 CMA Ontario – January 2008


Module 3 - Management Accounting

(this page left blank)

Page 4 CMA Ontario – January 2008


Module 3 - Management Accounting

1. Role of the Management Accountant & Ethics and the


Management Accountant

Exercise 1-21
The accountant's duties are often not sharply defined, so some of these answers might be
challenged.

a. Attention directing
b. Problem solving
c. Scorekeeping
d. Scorekeeping
e. Scorekeeping
f. Attention directing
g. Problem solving
h. Scorekeeping
i. Problem solving
j. Attention directing

Exercise 1-22

Report Statement Purpose


a Problem solving
b Scorekeeping
c Attention directing and scorekeeping
d Scorekeeping
e Scorekeeping

Exercise 1-24

The plan or budget communicates the financial goals the organization will achieve while
control arises from feedback on how well the plan has been achieved and reasons why
the plan has not been achieved.

1. Annual financial statements communicate what was achieved. The annual report
is a standardized control report on financial performance. It is feedback on what
the organization accomplished.

2. Internal periodic reports of financial performance are control reports.

3. Studies of new business development opportunities communicate planned costs


and revenue.

Page 5 CMA Ontario – January 2008


Module 3 - Management Accounting

4. Weekly reports of the total quantity of particular books sold are feedback. They
are control reports internally because a comparison can be made with the plan to
determine if the plan was achieved and if not, why not.

5. The report of losses suffered from a storm is a financial report that is a control
report. Externally the insurer will use the report to estimate the amounts it will
reimburse Indigo according to the insurance contract. Internally the managers
will use the report to modify their plan and generate the most appropriate
response to an unanticipated event.

The actual event will also initiate review of the adequacy of the insurance
coverage relative to its cost. This new data will be used in subsequent plans for
future insurance coverage and its cost.

Problem 1-25

1. Planning is choosing goals, predicting results under various ways of achieving


those goals, and then deciding how to attain the desired goals. One goal of DSN
is to increase operating income. Increasing revenues potentially is one way to
achieve this if the increase in revenues exceeds any associated increase in costs.
DSN expects daily circulation to increase from 250,000 per day in April to
400,000 per day in May. This budgeted circulation gain is expected to increase
newspaper revenues from $6,300,000 in April to a budgeted $7,440,000 in May.

Control covers both the action that implements the planning decision and the
performance evaluation of the personnel and operations. At DSN, the price drop
would be announced to its sales force and probably to customers. Requirement 2
illustrates a performance report for May 2007.

2. Actual Budgeted
Results Amounts Variance
Newspapers sold 13,640,000 12,400,000 1,240,000 F
Price per paper $0.60 $0.60 -0-
Newspaper revenue $8,184,000 $7,440,000 $744,000 F

3. Actions Campbell might take based on the $744,000 unfavourable variance for
circulation revenue include:

(a) Change predictions. DSN underestimated the daily circulation gain by


40,000 copies per day. It might examine the procedures it uses to estimate
the response of circulation to price changes.

(b) Change operations. DSN might now change its advertising rates to reflect
that circulation in May is 82% above that of April. This gives advertisers a
much larger audience they can reach with each advertisement in the DSN.

Page 6 CMA Ontario – January 2008


Module 3 - Management Accounting

Problem 1-26

1. Planning decisions
a. Decision to raise monthly subscription fee
c. Decision to upgrade content of online services
e. Decision to decrease monthly subscription fee

Control decisions
b. Decision to inform existing subscribers about the rate of increase-an
implementation part of control decisions
d. Dismissed the VP of Marketing-performance evaluation and feedback
aspect of control decisions

2. Planning decisions at WebNews.com focus on organizational goals, predicting


results under various alternative ways of achieving those goals, and then deciding
how to attain the desired goal, for example:
(a) WebNews.com could have the objective of revenue growth to gain critical
mass.
(b) it could have the objective of increasing operating income.

Many Internet companies in their formative years make revenue growth (and
subscriber growth) their primary goal.

Control focuses on deciding on, and taking actions that implement, the planning
decision, and deciding on performance evaluation and the related feedback that
will help future decision making. For example, WebNews.com will
(a) communicate the new rates to advertisers
(b) communicate the new price to marketing representatives

Problem 1-27

1. a. and e. Decisions to change subscription fee.


Problem solving - report outlining expected revenues from
subscribers and advertising with different monthly fee amounts.
Scorekeeping - report on number of monthly subscribers and
revenues generated in prior months.
Attention directing - report showing the change in the number of
subscribers of Internet companies at the time they change their
monthly fees.

b. Decision in June 2008 to inform existing subscribers of rate


increase in July.
Problem solving - report showing the cost of different ways of
informing subscribers of the rate increase.

Page 7 CMA Ontario – January 2008


Module 3 - Management Accounting

Scorekeeping - report showing how many subscribers immediately


paid the new subscription fee when past fee increases occurred.
Attention directing - report showing the number of subscribers to
the service that have not logged on for two months or more.

c. Decision to upgrade content of online services.


Problem solving - report showing the expected cost of alternative
ways to upgrade content.
Scorekeeping - labour cost tracking of software developers who
work on content.
Attention directing - report on cost overruns relative to budget for
ongoing content upgrades.

d. Dismissed the VP of Marketing.


Problem solving - budgeted cost of marketing department with
alternative management teams.
Scorekeeping - report showing breakdown of subscribers into
renewals and new subscribers, as well as the number of subscriber
cancellations.
Attention directing - report highlighting subscriber growth and
rates of competing Internet news services.

2. The VP of Marketing was demoted because actual subscribers fell short of


budgeted subscribers for each of three months, July through September 2006.

3. The company could also revise the expected or revised budgeted numbers of new
subscribers either downward or upward depending on how elastic they believe
demand is with respect to price.

Page 8 CMA Ontario – January 2008


Module 3 - Management Accounting

2. Cost Classifications

Exercise 2-18
1.
Supreme Deluxe Regular
Direct materials costs $ 100.80 $ 64.80 $ 74.40
Direct mfg labour costs 16.80 33.60 9.60
Indirect mfg. costs 50.40 100.80 28.80
Total mfg, costs $168.00 $199.20 $112.80

Kilograms produced 80 120 100


Cost per kilogram $2.100 $1.660 $1.128

2. Given the unit volume changes for August 2007, the use of unit costs from the
past month at a different unit volume level (both in aggregate and at the individual
product level) will yield incorrect estimates of total costs in August 2007.

Exercise 2-19

1. Direct costs are costs that are related to the particular paper products (Supreme,
Deluxe, or Regular) and can be traced to each one in an economically feasible
(cost-effective) way.

Indirect costs are costs that are related to the particular paper products (Supreme,
Deluxe, or Regular) but cannot be traced to each one in an economically feasible
(cost-effective) way.

2. Energy costs of $108 million can be traced to each individual production


line. This tracing will result in a more accurate assignment of costs to
products than when the $180 million of indirect manufacturing costs ($24
million of which is fixed) is allocated using direct manufacturing labour
costs at each line. The $108 million in energy costs does not have an
identical relationship to direct manufacturing labour costs for each
product line as is assumed when the direct manufacturing labour cost
allocation base is used in Question 2-18:

Supreme Deluxe Regular Total


1. Direct energy costs $47.76 $24.84 $35.40 $108.00
2. Direct manuf. labour cost 16.80 33.60 9.60 60.00
3. Ratio of 1 to 2 2.843 0.739 3.688 1.80

Page 9 CMA Ontario – January 2008


Module 3 - Management Accounting

The Supreme and the Regular product line present energy-intensive usage vis-à-
vis their direct manufacturing labour cost content. The result is that the product
lines will be undercosted when the Exercise 2-18 unit cost numbers are used.

3.
Supreme Deluxe Regular
Direct materials costs $ 100.80 $ 64.80 $ 74.40
Direct manuf. labour costs 16.80 33.60 9.60
Direct energy costs 47.76 24.84 35.40
Indirect manuf. costs 20.16 40.32 11.52
Total manuf. costs $185.52 $163.56 $130.92

Kilograms produced 80 120 100


Cost per kilogram $ 2.319 $ 1.363 $1.309

The unit cost amounts are:


Supreme Deluxe Regular
1. Exercise 2-19 $2.319 $1.363 $1.363
2. Exercise 2-18 2.100 1.660 1.128
Ratio of 1 to 2 1.1043 0.8211 1.1606

As predicted in requirement 2, the tracing of the higher than average energy costs
to Supreme and Regular results in an increase in reported unit costs for these
product lines.

Page 10 CMA Ontario – January 2008


Module 3 - Management Accounting

Exercise 2-21

1. (a) $120,000 ÷ 2,000 = $60.00 per package


(b) $120,000 ÷ 6,000 = $20.00 per package
(c) $120,000 ÷ 10,000 = $12.00 per package
(d) [$120,000 + (10,000 × $9.60)] ÷ 20,000
= $216,000 ÷ 20,000 = $10.80 per package

The unit cost to ECG decreases on a per-unit base due to the first $120,000 payment
being a fixed cost. The $9.60 amount per package beyond 10,000 units is a variable
cost. The cost function is:

$312,000

Total $216,000
Costs
$120,000
$120,000

10,000 20,000 30,000


Packages Sold

2. ECG should not use any of the unit costs in requirement 1 when predicting total
costs. Up to 10,000 units, the total cost is a fixed amount. Beyond 10,000 units,
the total cost is a combination of a fixed amount plus a per-unit (beyond 10,000
unit) variable amount. The total costs at different volume levels cannot be
predicted by using the unit cost at a specific volume level. The total cost should
be predicted by combining the total fixed costs and total variable costs rather than
multiplying a unit cost amount by the predicted number of packages sold.

Page 11 CMA Ontario – January 2008


Module 3 - Management Accounting

Exercise 2-25

Cost object: Each individual focus group

Cost variability: With respect to changes in the number of focus groups

There may be some debate over classifications of individual items. Debate is


more likely as regards cost variability.

Cost Item D or I V or F
A D V
B I F
C I Va
D I F
E D V
F I F
G D V
H I Vb

a Some students will note that phone call costs are variable when each call has a separate charge. It
may be a fixed cost if Consumer Focus has a flat monthly charge for a line, irrespective of the
amount of usage.
b Gasoline costs are likely to vary with the number of focus groups. However, vehicles likely serve
multiple purposes and detailed records may be required to examine how costs vary with changes
in one of the many purposes served.

Page 12 CMA Ontario – January 2008


Module 3 - Management Accounting

Exercise 2-29

(a) Canesco Company


Schedule of Cost of Goods Manufactured
for the year ended December 31, 2007 (‘000s)

Direct materials used


Direct materials inventory, January 1, 2007 $26,400
Direct materials purchased 90,000
Direct materials inventory, December 31, 2007 (31,200)
85,200
Direct manufacturing labour 30,000
Manufacturing overhead
Indirect manufacturing labour $18,000
Plant insurance 10,800
Amortization – plant building and equipment 13,200
Repairs and maintenance – plant 4,800 46,800
Total manufacturing costs 162,000
Work-in-process inventory, January 1, 2007 25,200
Work-in-process inventory, December 31, 2007 (24,000)
Cost of goods manufactured $163,200

(b) Canesco Company


Income Statement
for the year ended December 31, 2007 (‘000s)

Revenues $360,000
Cost of goods sold
Finished goods inventory, January 1, 2007 $21,600
Cost of goods manufactured 163,200
Finished goods inventory, December 31, 2007 (27,600) 157,200
Gross margin 202,800
Operating expenses
Marketing, distribution and customer service 111,600
General and administrative costs 34,800 146,400
Operating income $56,400

Page 13 CMA Ontario – January 2008


Module 3 - Management Accounting

Problem 2-32

Chan Corporation
Schedule of Cost of Goods Manufactured
for the year ended December 31, 2007 (in millions)

Direct materials used


Direct materials inventory, January 1, 2007 $36.00
Direct materials purchased 96.00
Direct materials inventory, December 31, 2007 (6.00)
126.00
Direct manufacturing labour 48.00
Manufacturing overhead
Indirect manufacturing labour $24.00
Plant utilities 6.00
Amortization – plant building and equipment 10.80
Plant supplies 7.20
Property taxes on plant 1.20
Miscellaneous 12.00 61.20
Total manufacturing costs 235.20
Work-in-process inventory, January 1, 2007 12.00
Work-in-process inventory, December 31, 2007 (2.40)
Cost of goods manufactured $244.80

Chan Corporation
Income Statement
for the year ended December 31, 2007 (in millions)

Revenues $420.00
Cost of goods sold
Finished goods inventory, January 1, 2007 $48.00
Cost of goods manufactured 244.80
Finished goods inventory, December 31, 2007 (14.40) 278.40
Gross margin 141.60
Marketing, distribution and customer service 108.00
Operating income $33.60

Page 14 CMA Ontario – January 2008


Module 3 - Management Accounting

Problem 2-33

1. The schedule in 2-31 can become a Schedule of Cost of Goods Manufactured and
Sold simply by including the beginning and ending finished goods inventory
figures in the supporting schedule, rather than directly in the body of the income
statement. Note that the term cost of goods manufactured refers to the cost of
goods brought to completion (finished) during the accounting period, whether they
were started before or during the current accounting period. Some of the
manufacturing costs incurred are held back as costs of the ending work in process;
similarly, the costs of the beginning work in process inventory become a part of
the cost of goods manufactured for 2007.

2. The sales manager’s salary would be charged as a marketing cost as incurred by


both manufacturing and merchandising companies. It is basically an operating cost
that appears below the gross margin line on an income statement. In contrast, an
assembler’s wages would be assigned to the products worked on. Thus, the wages
cost would be charged to Work in Process and would not be expensed until the
product is transferred through Finished Goods Inventory to Cost of Goods Sold as
the product is sold.

3. The direct-indirect distinction can be resolved only with respect to a particular cost
object. For example, in defence contracting, the cost object may be defined as a
contract. Then, a plant supervisor’s salary may be charged directly and wholly to
that single contract.

4. Direct materials used = $126,000,000 / 1,000,000 units = $126.00 per unit


Amortization = $10,800,000 / 1,000,000 units = $10.80 per unit

5. Direct materials unit cost would be unchanged at $126. Amortization unit cost
would be $10,800,000 ÷ 1,500,000 = $7.20 per unit. Total direct materials costs
would rise by 50% to $189,000,000 ($126 × 1,500,000). Total amortization cost of
$10,800,000 would remain unchanged.

6. Unit costs are averages, and they must be interpreted with caution. The $126 direct
materials unit cost is valid for predicting total costs because direct materials is a
variable cost; total direct materials costs indeed change as output levels change.
However, fixed costs like amortization must be interpreted quite differently from
variable costs. A common error in cost analysis is to regard all unit costs as one—
as if all the total costs to which they are related are variable costs. Changes in
output levels (the denominator) will affect total variable costs, but not total fixed
costs. Graphs of the two costs may clarify this point; it is safer to think in terms of
total costs rather than in terms of unit costs.

Page 15 CMA Ontario – January 2008


Module 3 - Management Accounting

Problem 2-35

When solving a problem like this one, the best approach is usually to put down the
numbers you have, and then solve the unknowns. In the table below the numbers
provided are in italic. The numbers beside the solved numbers represent the order in
which they were solved. Note that we do not need the Accounts Receivable or Payable
balances, so these are excluded.

Case 1 Case 2
Cost of goods manufactured -
Direct materials used $9,600 $14,400
Direct manufacturing labour 3,600 6,000
Indirect manufacturing costs 8,400 7,800
Total manufacturing costs 21,600 2 28,200 4
WIP – January 1, 2007 0 960
WIP – December 31, 2007 0 (3,600)
Cost of goods manufactured 21,600 3 $25,560 3

Revenues $38,400 $38,160


Cost of goods sold
Finished goods inventory, January 1, 2007 4,800 4,800
Cost of goods manufactured 21,600 4 25,560 2
Finished goods inventory, December 31, 2007 (1,560) 5 (6,360)
24,840 1 24,000
Gross margin $13,560 $14,160 1

A = $24,840
B = $1,560
C = $14,160
D = $7,800

Page 16 CMA Ontario – January 2008


Module 3 - Management Accounting

Problem 2-40

In the table below the numbers provided are in italic. The numbers beside the solved
numbers represent the order in which they were solved and also corresponds to the
required.

Shaheen Plastics Inc.


Schedule of Cost of Goods Manufactured
for the month ended August 31, 2007 (in millions)

Direct materials used


Direct materials inventory, August 1, 2007 108
Direct materials purchased 432
Direct materials inventory, August 31, 2007 1 (90)
450
Direct manufacturing labour 3 894
Manufacturing overhead
Variable $300
Fixed 2 276 576
Total manufacturing costs 1,920
Work-in-process inventory, August 1, 2007 240
Work-in-process inventory, August 31, 2007 4 (180)
Cost of goods manufactured $1,980

Shaheen Plastics Inc.


Schedule of Cost of Goods Sold
for the month ended August 31, 2007 (in millions)

Finished goods inventory, August 1, 2007 $ 150


Cost of goods manufactured 1,980
5 2,130
Finished goods inventory, August 31, 2007 6 (90)
$2,040

Page 17 CMA Ontario – January 2008


Module 3 - Management Accounting

3. Job Order Costing


Problem 4-18

1. The overview diagram required is not necessary.


Machinery Department POR = $2,160,000 / 60,000 = $36.00 per MH
Assembly Department POR = $4,320,000 / 2,400,000 = $1.80 per DL$

2. Overhead applied:
Machining: 2,400 MH x $36 $86,400
Assembly: 18,000 DLH x $1.80 32,400
$118,800

3. Machining Assembly
Actual overhead $2,520,000 $4,440,000
Overhead applied
Machining: 66,000 MH x $36 2,376,000
Assembly: $2,640,000 DLH x $1.80 4,752,000
Over (Under) applied overhead ($144,000) $312,000

Problem 4-21

1. The overview diagram required is not necessary.

2. a. Raw materials inventory $960


Accounts payable $960

b. WIP 852
Raw materials inventory 852

c. Manufacturing overhead 120


Raw materials inventory 120

d. WIP 1,560
Wages payable 1,560

e. Manufacturing overhead 1,080


Wages payable 1,080

f. Manufacturing overhead 480


Accumulated amortization 480

g. Manufacturing overhead 660


Accounts payable 660

Page 18 CMA Ontario – January 2008


Module 3 - Management Accounting

h. WIP ($1,560 x 1.6) 2,496


Manufacturing Overhead 2,496

i. Finished Goods Inventory 4,944


WIP 4,944

j. Accounts receivable 9,600


Revenues 9,600

k. Cost of goods sold 4,824


Finished Goods Inventory 4,824

l. Manufacturing Overhead 156


Cost of goods sold 156

3. Raw Materials Inventory Work-in-Process


B $120 B $72
a. 960 852 b. b. 852 4,944 i.
120 c. d. 1,560
E $108 h. 2,496
E $36

Finished Goods Inventory Manufacturing Overhead


B $600 c. $120
i. 4,944 4,824 k. e. 1,080 2,496 h.
E $720 f. 480
g. 660
l. 156
E -0-

Cost of Goods Sold


k. $4,824 156 l.
E $4,668

Page 19 CMA Ontario – January 2008


Module 3 - Management Accounting

Problem 4-31

1. Direct Materials
Balance, December 30, 2007 ($120,000 – 84,000) $36,000

Work-in-Process
Balance, December 30, 2007 ($384,000 – 366,000) $18,000
Costs incurred December 31, 2007:
Direct labour 6,000
Manufacturing overhead applied: $6,000 x 60%* 3,600
$27,600

Finished Goods
Balance, December 30, 2007 ($390,000 – 360,000) $30,000

* POR = $108,000 Overhead Applied / 180,000 Direct Labour = 60%

2. Manufacturing overhead account -


Balance, December 30, 2007 ($102,000 – 108,000) $6,000 cr.
Manufacturing overhead incurred on Dec 31, 2007 -
Indirect labour 1,200 dr.
Miscellaneous 1,200 dr.
Manufacturing overhead applied on Dec 31, 2007 3,600 cr.
Balance, December 31, 2007 $7,200 cr.

WIP $6,000
Wages payable $6,000

Manufacturing overhead 1,200


Wages payable 1,200

Manufacturing overhead 1,200


Accounts payable 1,200

WIP 3,600
Manufacturing overhead 3,600

Manufacturing overhead 7,200


Cost of goods sold 7.200

Page 20 CMA Ontario – January 2008


Module 3 - Management Accounting

Problem 4-32

1. Not required.

2. $124,800 / 1,600 = $78 per hour

3. Direct labour hours = 25 professionals x 1,600 = 40,000


POR = $2,640,000 / 40,000 = $66 per hour

4. a. 100 hours x ($78 + 66) = $14,400


b. 150 hours x ($78 + 66) = $21,600

Problem 4-33

1. Professional partners: $240,000 / 1,600 = $150


Professional managers: $96,000 / 1,600 = $60

2. General support = $2,160,000 / 40,000 = $54


Secretarial support = $480,000 / 8,000 = $60

3. Richardson Inc. -
Professional partners: 60 hours x ($150 + 54 + 60) $15,840
Professional managers: 40 hours x ($60 + 54) 4,560
$20,400

Punch Inc. -
Professional partners: 30 hours x ($150 + 54 + 60) $7,920
Professional managers: 120 hours x ($60 + 54) 13,680
$21,600

4. The Richardson and Punch jobs differ in their use of resources. The Richardson
job has a mix of 60% partners and 40% associates, while Punch has a mix of 20%
partners and 80% associates. Thus, the Richardson job is a relatively high user of
the more costly partner-related resources (both direct partner costs and indirect
partner secretarial support). The refined-costing system in Problem 4-33 increases
the reported cost in Problem 4-32 for the Richardson job by 41.7% (from $14,400
to $20,400).

Page 21 CMA Ontario – January 2008


Module 3 - Management Accounting

Problem 4-36

1. Machining: $7,200,000 / 100,000 = $72 per machine hour


Assembly: $6,000,000 / 125,000 = $48 per direct labour hour

2. Machining Assembly
Actual manufacturing overhead $7,440,000 $5,640,000
Applied manufacturing overhead
90,000 machine hours x $72 6,480,000
120,000 direct labour hours x $48 5,760,000
Over (Under)-applied overhead ($960,000) $120,000

a. COGS $960,000
Manufacturing Overhead – Machining $960,000

Manufacturing Overhead – Assembly 120,000


COGS 120,000

b. Proration - Proration -
Balance % Machining Assembly
COGS $19,200,000 80.00% $768,000 $96,000
Finished goods 900,000 3.75% 36,000 4,500
WIP 3,900,000 16.25% 156,000 19,500
$24,000,000 $960,000 $120,000

COGS $768,000
Finished goods 36,000
WIP 156,000
Manufacturing Overhead – Machining $960,000

Manufacturing Overhead – Assembly 120,000


COGS 96,000
Finished goods 4,500
WIP 19,500

c. Machining -
Allocated
Overhead % Proration
COGS: 67,500 x $72 $4,860,000 75% $720,000
Finished goods: 4,500 x $72 324,000 5% 48,000
WIP: $18,000 x $72 1,296,000 20% 192,000
$6,480,000 $960,000

Page 22 CMA Ontario – January 2008


Module 3 - Management Accounting

COGS $720,000
Finished goods 48,000
WIP 192,000
Manufacturing Overhead – Machining $960,000

Assembly -
Allocated
Overhead % Proration
COGS – 90,000 x $48 $4,320,000 75% $90,000
Finished goods – 4,800 x $48 230,400 4% 4,800
WIP – 25,200 x $48 1,209,600 21% 25,200
$5,760,000 $120,000

Manufacturing Overhead – Assembly $120,000


COGS $90,000
Finished goods 4,800
WIP 25,200

3. If the purpose is to report the most accurate inventory and cost of goods sold
figures, the preferred method is to prorate based on the manufacturing overhead
allocated amount in the inventory and cost of goods sold accounts (as in
requirement 2c). Note, however, that prorating based on ending balances in Work
in Process, Finished Goods, and Cost of Goods Sold (as in requirement 2b) yields a
close approximation to the more accurate proration in requirement 2c. Also note
that the write-off to Cost of Goods Sold method (as in requirement 2a) results in
account balances in Work in Process, Finished Goods, and Cost of Goods Sold that
are not very different from the most accurate method. Furthermore, the Write Off to
Cost of Goods Sold method is simpler than the other methods. Depending on the
objectives of proration, a manager may prefer any one of the methods over the
other two.

Page 23 CMA Ontario – January 2008


Module 3 - Management Accounting

Problem 4-40

1. Credit to the Materials Control account = $456,000

2. Direct manufacturing labour hours = $432,000 / $18 = 24,000


Manufacturing overhead applied = 24,000 hours x $24 = $576,000

3. Cost of goods manufactured = Debit to the Finished Goods Inventory account =


$1,128,000

4. Balance
WIP Account balance, January 1, 2007 $ 24,000
Direct manufacturing labour 432,000
Direct materials 456,000
Manufacturing overhead applied 576,000
Less Cost of goods manufactured (1,128,000)
WIP Account balance, December 31, 2007 $360,000

5. COGS = Credit to the Finished Goods Inventory account = $1,080,000

6. Under applied overhead = $648,000 Actual – 576,000 Applied = $72,000

7. a. COGS $72,000
Manufacturing overhead $72,000

b. Balance % Proration
COGS ($1,080,000 – 24,000 Opening WIP
– 12,000 Opening FG) $1,044,000 71.3% $51,336
WIP 360,000 24.6% 17,712
FG ($12,000 + 1,128,000 – 1,080,000) 60,000 4.1% 2,952
$1,464,000 $72,000

COGS $51,336
WIP Inventory 17,712
FG Inventory 2,952
Manufacturing overhead $72,000

8. Operating income before effect of proration -


$1,308,000 Sales – 1,080,000 COGS – 168,000 Marketing Expenses = $60,000

Operating income if under applied overhead is written off to COGS


$60,000 - 72,000 = ($12,000)

Operating income if under applied overhead is prorated


$60,000 - 51,336 = $8,664

Page 24 CMA Ontario – January 2008


Module 3 - Management Accounting

9. If the purpose is to report the most accurate inventory and cost of goods sold
figures, the preferred method is to prorate based on the manufacturing overhead
allocated component in the Inventory and Cost of Goods Sold accounts. Proration
based on the balances in Work in Process, Finished Goods, and Cost of Goods
Sold will equal the proration based on the manufacturing overhead allocated
component if the proportions of direct costs to manufacturing overhead costs are
constant in the Work in Process, Finished Goods, and Cost of Goods Sold
accounts. Even if this is not the case, the prorations based on Work in Process,
Finished Goods, and Cost of Goods Sold will better approximate the results if
actual cost rates have been used than the write-off to Cost of Goods Sold method.
Another consideration in Needham’s decision about how to dispose of
underallocated manufacturing overhead is the effects on operating income. The
write-off to Cost of Goods Sold will lead to an operating loss. Proration based on
the balances in Work in Process, Finished Goods, and Cost of Goods Sold will
help Needham avoid the loss and show an operating income.
The main merit of the write-off to cost of goods sold method is its simplicity.
However, accuracy and the effect on operating income favour the preferred and
recommended proration approach.

Page 25 CMA Ontario – January 2008


Module 3 - Management Accounting

4. Process Costing and Spoilage

Problem 17-22 / 17-23

Direct Conversion
Equivalent Units - Materials Costs
Transferred out 51.0 51.0
WIP - end
12 x 60% 7.2
12 x 30% 3.6
58.2 54.6

Costs
WIP - beginning $ 5,426,960 $ 1,001,440
Costs Added 35,420,000 15,312,000
$40,846,960 $16,313,440

Unit Cost $701,838 $298,781

Total unit cost = $1,000,619

Units transferred out: 51 x $1,000,619 $51,031,569

Units in ending WIP


Direct materials:7.2 x $701,838 $5,053,234
Conversion costs: 3.6 x $298,781 1,075,612
$6,128,846

Work-in-Process Finished Goods


B $ 6,428,400 51,031,569 TO 51,031,569
CA 50,732,000
E 6,128,831

Difference due to rounding.

Page 26 CMA Ontario – January 2008


Module 3 - Management Accounting

Problem 17-24 / 17-25

Direct Conversion
Equivalent Units - Materials Costs
Transferred out
From Opening WIP
8 x 10% | 8 x 60% 0.8 4.8
Started and Completed: 51 – 8 43.0 43.0
WIP - end 7.2 3.6
51.0 51.4

Costs Added $35,420,000 $15,312,000

Unit Cost $992,409 $694,510 $297,899

Cost of units transferred out -


WIP - beginning
Costs in opening WIP $6,428,400
Cost to complete
Direct materials: 0.8 x $694,510 555,608
Conversion costs: 4.8 x $297,899 1,429,915

Units Started and Completed - 43 x $992,409 42,673,587


$51,087,510

Units in ending WIP


Direct materials: 7.2 x $694,510 $5,000,472
Conversion costs: 3.6 x $297,899 1,072,436
$6,072,908

Work-in-Process Finished Goods


B $ 6,428,400 51,087,510 TO 51,087,510
CA 50,732,000
E 6,072,890

Difference due to rounding.

Page 27 CMA Ontario – January 2008


Module 3 - Management Accounting

Problem 17-30 / 17-31

Direct Conversion
Accounted for Materials Costs
Transferred out 95.0 95.0
WIP – end: 15 x 100% | 15 x 70% 15.0 10.5
110.0 105.5

Costs
WIP - beginning $ 506,000 $ 132,000
Costs Added 2,200,000 1,028,500
$2,706,000 $1,160,500

Unit Cost $35,600 $24,600 $11,000

Cost of goods transferred out: 95 x 35,600 $3,382,000

Units in ending WIP


Direct materials: 15 x 24,600 $369,000
Conversion costs: 10.5 x 11,000 115,500
$484,500

WIP - Assembly WIP - Testing


B $638,000 3,382,000 TO 3,382,000
CA 3,228,500
E 484,500

Page 28 CMA Ontario – January 2008


Module 3 - Management Accounting

Problem 17-32

Direct Conversion
Equivalent Units - Materials Costs
Transferred out
From Opening WIP: 25 x 0% | 25 x 40% 0.0 10.0
Started and Completed: 95 - 25 70.0 70.0
WIP – end: 15 x 100% | 15 x 70% 15.0 10.5
85.0 90.5

Costs Added $2,200,000 $1,028,500

Unit Cost $37,246.95 $25,882.35 $11,364.64

Cost of goods transferred out -


WIP - beginning
Costs in opening WIP $638,000
Cost to complete
Conversion costs: 10 x $11,364.64 113,646

Units Started and Completed - 70 x $37,246.95 2,607,287


$3,358,933
Units in ending WIP
Direct materials: 15 x $25,882.35 $388,235
Conversion costs: 10.5 x $11,364.64 119,329
$507,564

WIP - Assembly WIP - Testing


B $638,000 3,358,933 TO 3,358,933
CA 3,228,500
E 506,567

Difference due to rounding.

Page 29 CMA Ontario – January 2008


Module 3 - Management Accounting

Problem 17-33

Transferred Direct Conversion


Equivalent Units in costs Materials Costs
Transferred out 110 110 110
WIP – end: 20 x 100% | 20 x 0% | 20 x 60% 20 0 12
130 110 122

Costs -
WIP - beginning $1,084,380 $ 0 $ 364,980
Costs Added 3,382,000 3,885,000 1,581,000
$4,466,380 $3,885,000 $1,945,980

Unit Cost $85,625.61 $34,356.77 $35,318.18 $15,950.66

Cost of goods manufactured: 110 x $85,625.61 $9,418,817

Units in ending WIP


Transferred in Costs: 20 x $34,356.77 687,135
Conversion costs: 12 x $15,950.66 191,408
$878,543

Work-in-Process Finished Goods


B $1,449,360 9,418,817 TO 9,418,817
TI 3,382,000
CA 5,466,000
E 878,543

Page 30 CMA Ontario – January 2008


Module 3 - Management Accounting

Problem 17-34

Transferred Direct Conversion


Equivalent Units in costs Materials Costs
Transferred out
From Opening WIP: 35 x 0% | 35 x 100%
35 x 30% 0.0 35.0 10.5
Started and Completed: 110 – 35 75.0 75.0 75.0
WIP – end: 20 x 100% | 20 x 0% | 20 x 60% 20.0 0.0 12.0
95.0 110.0 97.5

Costs Added $3,358,936 $3,885,000 $1,581,000

Unit Cost $86,890.78 $35,357.22 $35,318.18 $16,215.38

Cost of goods manufactured -


WIP - beginning
Costs in opening WIP $1,443,046
Cost to complete
Direct materials: 35 x 35,318.18 1,236,136
Conversion costs: 10.5 x 16,215.38 170,261

Units Started and Completed – 75 x 86,890.78 6,516,809


$9,366,252

Units in ending WIP


Transferred in costs: 20 x 35,357.22 $707,144
Conversion costs: 12 x 16,215.38 194,585
$901,729

Work-in-Process Finished Goods


B $1,443,046 9,366,252 TO 9,366,252
TI 3,358,936
CA 5,466,000
E 901,730

Page 31 CMA Ontario – January 2008


Module 3 - Management Accounting

Problem 18-24 to 18-27

FIFO
WIP Abnormal Spoilage Finished Goods
B 3.060 45,684 252 45,684
C 47,916 252 AS

E 5,040

B = Beginning Balance E = Ending Balance AS = Abnormal Spoilage


C = Costs Added

Equivalent Units: Direct Conversion


Materials Costs
Units Transferred Out:
From opening inventory: 1,000 x 0% | 1,000 x 50% 0 500
Units Started and Completed: 9,000 – 1,000 8,000 8,000
Normal Spoilage 100 100
Abnormal Spoilage 50 50
Units in ending WIP 2,000 600
10,150 9,250

Unit cost Calculations:


Direct materials: $14,616 / 10,150 $1.44
Conversion costs: $33,300 / 9,250 3.60
$5.04

Cost of Good Units Transferred Out:


Opening WIP:
Costs in opening WIP $3,060
Costs to complete: 500 x $3.60 1,800

Units Started and completed:


(8,000 Units S+C + 100 Normal Spoilage) x $5.04 40,824
$45,684

Abnormal Spoilage: 50 x $5.04 $252

Cost of Ending WIP (Proof)


DM: 2,000 x $1.44 $2,880
CC: 600 x $3.60 2,160
$5,040

Page 32 CMA Ontario – January 2008


Module 3 - Management Accounting

Weighted Average

WIP Abnormal Spoilage Finished Goods


B 3.060 45,682 251 45,682
C 47,916 251 AS

E 5,043

B = Beginning Balance AS = Abnormal Spoilage


C = Costs Added E = Ending Balance

Equivalent Units: Direct Conversion


Materials Costs
Units Transferred Out: 9,000 9,000
Normal Spoilage 100 100
Abnormal Spoilage 50 50
Units in ending WIP 2,000 600
11,150 9,750

Unit cost Calculations:


Direct materials: ($1,560 + $14,616) / 11,150 $1.45076
Conversion costs: ($1,500 + $33,300) ÷ 9,750 3.56923
$5.01999

Cost of Units Transferred Out:


(9,000 Units TO + 100 Normal Spoilage) x $5.01999 $45,682

Abnormal Spoilage: 50 x $5.01999 $251

Cost of Ending WIP (Proof)


DM: 2,000 x $1.45076 $2,902
CC: 600 x $3.56923 2,142
$5,044

Page 33 CMA Ontario – January 2008


Module 3 - Management Accounting

Problem 18-30

Weighted Average

WIP Abnormal Spoilage Finished Goods


B 504,000 1,040,520 45,240 1,040,520
C 1,203,840 45,240 AS

E 622,080

B = Beginning Balance AS = Abnormal Spoilage


C = Costs Added E = Ending Balance

Equivalent Units: Direct Conversion


Materials Costs
Units Transferred Out: 40,000 40,000
Normal Spoilage 6,000 6,000
Abnormal Spoilage 2,000 2,000
Units in ending WIP: 32,000 x 100% | 32,000 x 75% 32,000 24,000
80,000 72,000

Unit cost Calculations:


Direct materials: ($288,000 + 504,000) ÷ 80,000 $ 9.90
Conversion costs: ($216,000 + 699,840) ÷ 72,000 12.72
$22.62

Cost of Units Transferred Out:


(40,000 + 6,000 Normal Spoilage) x $22.62 $1,040,520

Abnormal Spoilage: 2,000 x $22.62 $45,240

Cost of Ending WIP (Proof)


DM: 32,000 x $9.90 $316,800
CC: 24,000 x $12.72 305,280
$622,080

Page 34 CMA Ontario – January 2008


Module 3 - Management Accounting

Problem 18-31

FIFO
WIP Abnormal Spoilage Finished Goods
B 504,000 1,028,160 46,080 1,028,160
C 1,203,840 46,080 AS

E 633,600

B = Beginning Balance E = Ending Balance AS = Abnormal Spoilage


C = Costs Added

Equivalent Units: Direct Conversion


Materials Costs
Units Transferred Out:
From op. inventory: 30,000 x 0%| 30,000 x 40% 0 12,000
Units Started and Completed: 40,000 – 30,000 10,000 10,000
Normal Spoilage 6,000 6,000
Abnormal Spoilage 2,000 2,000
Units in ending WIP: 32,000 x 100% | 32,000 x 75% 32,000 24,000
50,000 54,000

Unit cost Calculations:


Direct materials: $504,000 ÷ 50,000 $10.08
Conversion costs: $699,840 ÷ 54,000 12.96
$23.04

Cost of Good Units Transferred Out:


Opening WIP:
Costs in opening WIP $504,000
Costs to complete: 12,000 x $12.96 155,520

Units Started and completed:


(10,000 + 6,000 Normal Spoilage) x $23.04 368,640
$1,028,160

Abnormal Spoilage: 2,000 x $23.04 $46,080

Cost of Ending WIP (Proof)


DM: 32,000 x $10.08 $322,560
CC: 24,000 x $12.96 311,040
$633,600

Page 35 CMA Ontario – January 2008


Module 3 - Management Accounting

Problem 18-39

1. Raw materials inventory $8,400


Materials-related manufacturing overhead $8,400

2. Cash or A/R 8,400


Raw materials inventory 8.400

3. Total materials-related manufacturing overhead allocated to the two products:


= (20,000 x $2.40) + (10,000 x $3.60)
= $48,000 + 36,000 = $84,000

Allocation of scrap:
HM3: $8,400 x 48,000 / 84,000 = $4,800
JB4: $8,400 x 36,000 / 84,000 = $3,600

Revised manufacturing unit costs:


HM3 JB4
Direct materials, direct manufacturing labour
and other manufacturing overhead $22.80 $32.40
Materials-related manufacturing overhead -
($48,000 – 4,800) / 20,000 2.16
($36,000 – 3,600) / 10,000 3.24
$24.96 $35.64

Page 36 CMA Ontario – January 2008


Module 3 - Management Accounting

5. Cost Behaviour

Regression results for Exercise 10-25

Regression Statistics
R Square 96.2%
Standard Error 1,737.60
Observations 12

Standard
Coefficients Error t Stat
Intercept 57,924.74 1,498.46 38.65620
Labour Hours 4.71 0.29468 15.98801

Exercise 10-19

1. (1)
2. (7) A step-cost function rather than a fixed cost
3. (11)
4. (2)
5. (9)
6. (12)
7. (3)
8. (9)

Page 37 CMA Ontario – January 2008


Module 3 - Management Accounting

Exercise 10-22

1.

There is a positive relationship between the number of service reports (the cost driver)
and customer service department cost. The relationship is economically plausible.

2. Customer
Number of Service
Service Department
Reports Costs
High 436 $26,268
Low 122 15,530
Difference 314 $10,738

Variable cost = $10,738 / 314 = $34.197 per service report

Fixed costs = $26,268 – (436 x $34.197) = $11,358

Customer service department costs = $11,358 + $34.197X


Where X = number of service reports

3. Other possible cost drivers of customer service department costs are:


• Number of products replaced with a new product (and the dollar value of the new
products charged to the customer service department).
• Number of products repaired and the time and cost of repairs.

Page 38 CMA Ontario – January 2008


Module 3 - Management Accounting

Exercise 10-25

1.

The regression line is:

Y = $57,925 + $4.71X

Economic plausibility. Labour-hours appears to be an economically plausible driver of


overhead costs for a catering company. Overhead costs such as scheduling, hiring,
training of workers, and managing the workforce are largely incurred to support labour.

Goodness of fit. The r2 = 96.2% indicating that labour hours explain 96.2% of the
variation in overhead costs. The good fit indicates a strong relationship between the
labour-hour cost driver and overhead costs.

Slope of regression line. The t-value of 16 is much greater than 2.0 (the rule of thumb)
indicating that the probability that the line is horizontal is very low.

2. Food and beverages $18.00


Labour (0.5 hour x $12.00 per hour) 6.00
Variable overhead (0.5 hour x $4.71 per hour) 2.36
$26.36

Page 39 CMA Ontario – January 2008


Module 3 - Management Accounting

3. To earn a positive contribution margin, the minimum bid for a 200-person cocktail
party would be any amount greater than $5,272. This amount is calculated by
multiplying the variable cost per person of $26.36 by the 200 people. At a price above
the variable costs of $5,272 Bob Jones will be earning a contribution margin toward
coverage of his fixed costs.

Of course, Bob Jones will consider other factors in developing his bid including (a) an
analysis of the competition—vigorous competition will limit Jones's ability to obtain a
higher price; (b) a determination of whether or not his bid will set a precedent for lower
prices––overall, the prices Bob Jones charges should generate enough contribution to
cover fixed costs and earn a reasonable profit; (c) a judgment of how representative past
historical data (used in the regression analysis) is about future costs.

Page 40 CMA Ontario – January 2008


Module 3 - Management Accounting

Problem 10-35

1.

Page 41 CMA Ontario – January 2008


Module 3 - Management Accounting

2. Regression 1 Regression 2
Independent Variable = Independent Variable =
Criteria # of Academic Programs Enrolled Students

Economic Plausibility A positive relationship A positive relationship


between overhead costs between overhead costs
and number of academic and number of enrolled
programs is economically students is economically
plausible at Eastern plausible at Eastern
University. University.

Goodness of Fit r2 = 72% r2 = 55%


Excellent fit. Good fit, but not as good as
Regression1

Significance of t-value = 5.08; significant t-value = 3.52; significant


independent variable

Both regression models appear to perform well when estimating overhead costs.
Regression 1, using the number of academic programs as the independent variable,
appears to perform slightly better than Regression 2, which uses number of enrolled
students as the independent variable. Regression 1 has a high r2 and goodness of fit, a
high t-value, indicating a significant relationship between number of academic
programs and overhead costs. Regression 2 has a lower r2 than Regression 1.

3. The analysis indicates that overhead costs are related to the number of academic
programs and the number of enrolled students. If Eastern has pressures to reduce and
control overhead costs, it may need to look hard at closing down some of its academic
programs and reducing its intake of students. Reducing the number of enrolled students
may cut down on overhead costs, but it also cuts down on revenues (tuition payments),
hurts the reputation of the school, and reduces its alumni base, which is a future source
of funds. For these reasons, Eastern may prefer to downsize its academic programs,
particularly those programs that attract few students. Of course, Eastern should continue
to reduce costs by improving the efficiency of the delivery of its programs.

Page 42 CMA Ontario – January 2008


Module 3 - Management Accounting

Problem 10-36

1. It is economically plausible that the correct form of the model of overhead costs
includes both number of academic programs and number of enrolled students as cost
drivers. The findings in Problem 10-35 indicate that each of the independent variables
affects overhead costs. (Each regression has a significant r2 and t-value on the
independent variable.) Hanks could choose to divide overhead costs into two cost
pools, (i) those overhead costs that are more closely related to number of academic
programs and (ii) those overhead costs more closely related to number of enrolled
students, and rerun the simple regression analysis on each overhead cost pool.
Alternatively, Hanks could run a multiple regression analysis with total overhead costs
as the dependent variable and the number of academic programs and number of
enrolled students as the two independent variables.

2. Regression 3
Independent Variables = # of academic programs and
Criteria enrolled students

Economic Plausibility A positive relationship between overhead costs and


number of academic programs and number of enrolled
students is economically plausible at Eastern University.

Goodness of fit r2 = 81%, Adj r2 = 76.6%


Excellent fit.

Significance of # of academic programs, t-value = 3.46


independent variables enrolled students, t-value = 2.02
Both are significant.

Hanks should use the multiple regression model rather than the two simple regression
models of Problem 10-35. The multiple regression model appears economically
plausible and the regression model performs very well when estimating overhead costs.
It has an excellent goodness of fit, significant t-values on both independent variables,
and meets all the specification assumptions for ordinary least squares regression.

3. Possible uses for the multiple regression results include:


• Planning and budgeting at Eastern University. The regression analysis indicates
the variables (number of academic programs and number of enrolled students)
that help predict changes in overhead costs.
• Cost control and performance evaluation. Hanks could compare actual
performance with budgeted or expected numbers and seek ways to improve the
efficiency of the university operations, and evaluate the performance of managers
responsible for controlling overhead costs.
• Cost management. If cost pressures increase, the University could save costs by
closing down academic programs that have few students enrolled.

Page 43 CMA Ontario – January 2008


Module 3 - Management Accounting

6. Activity Based Costing

Exercise 5-17

1. Output unit-level costs—resources sacrificed on activities performed on each


individual unit of a product. The cost of these activities increases with each
additional unit of output produced.
c. Direct materials costs, $7,200,000
e. Direct manufacturing labour costs, $1,200,000
f. Machine-related overhead costs (amortization, maintenance, production-
engineering), $1,320,000.

Batch-level costs—resources sacrificed on activities that are related to a group of


units of a product rather than each individual unit of a product.
b. Purchase order-related costs (including receiving materials and paying
suppliers), $600,000.
d. Setup costs, $720,000

Product-sustaining costs—resources sacrificed on activities undertaken to support


individual products regardless of the number of units or batches in which the
product is produced.
a. Designing processes, drawing process charts, and making engineering
changes for individual products, $960,000.

Facility-sustaining costs—resources sacrificed on activities that cannot be traced to


individual products or services but support the organization as a whole.
g. Plant management, plant rent, and insurance, $1,080,000.

2. The complex boom box made in many batches will use significant batch-level and
product-sustaining overhead resources compared with the simple boom box that is
made in few batches. Because each boom box requires the same amount of
machine-hours, both the simple and the complex boom box will be allocated the
same amount of overhead costs per boom box if Telecom uses only machine-hours
to allocate overhead costs to boom boxes. As a result, the complex boom box will
be undercosted (it consumes a relatively high level of resources but is reported to
have a relatively low cost) and the simple boom box will be overcosted (it
consumes a relatively low level of resources but is reported to have a relatively high
cost).

Page 44 CMA Ontario – January 2008


Module 3 - Management Accounting

3. Using the cost hierarchy to calculate activity-based costs can help Telecom to
identify both the costs of individual activities and the costs of activities demanded
by individual products. Telecom can use this information to manage its business in
several ways:

• Pricing and product mix decisions. Knowing the resources needed to


manufacture and sell different types of boom boxes can help Telecom to
price the different boom boxes and also identify which boom boxes are more
profitable. It can then emphasize its more profitable products.
• Telecom can use information about the costs of different activities to
improve processes and reduce costs of the different activities. Telecom could
have a target of reducing costs of activities (setups, order processing, etc.)
by, say, 3% and could constantly seek to eliminate activities and costs (such
as engineering changes) that its customers perceive as not adding value
• Telecom management can identify and evaluate new designs to improve
performance by analyzing how product and process designs affect activities
and costs.
• Telecom can use its ABC systems and cost hierarchy information to plan and
manage activities. What activities should be performed in the period and at
what cost?

Exercise 5-21

1. POR = $370,320 / 4,000 = $92.58 per machine hour

2. United Motors: 120 machine hours x $92.58 $11,110


Holden Motors: 2,800 machine hours x $92.58 259,224
Leland Vehicle: 1,080 machine hours x $92.58 99,986

3. The conditions that would enable machine-hours to provide an accurate estimate of


variable manufacturing overhead cost incurred on each individual contract are:
a. machine-hours is the sole cost driver, and there is a linear relation between
variable manufacturing overhead cost and machine-hours, and
b. machine-hours used per contract can be accurately measured.

Page 45 CMA Ontario – January 2008


Module 3 - Management Accounting

Exercise 5-22

1. Design: $46,800 / 390 = $120 per design hour


Engineering: $35,520 / 370 = $96 per engineering hour
Production: $288,000 / 4,000 = $72 per machine hour

2. United Holden Leland


Design: $120 x 110 | 200 | 80 $13,200 $ 24,000 $ 9,600
Engineering: $96 x 70 | 60 | 240 6,720 5,760 23,040
Production: $72 x 120 | 2,800 | 1,080 8,640 201,600 77,760
$28,560 $231,360 $110,400

3. United Holden Leland


Cost allocation using…
Plantwide rate $11,110 $259,224 $99,986
ABC 28,560 231,360 $110,400

The three contracts differ sizably in the way they use the resources of the three
departments. The percentage of total driver units in each department is:

United Holden Leland


Design 28% 51% 21%
Engineering 19% 16% 65%
Production 3% 70% 27%

Whereby, the plantwide system allocated overhead based on machine hours based
on the following percentages:

United Holden Leland


Machine hours 3% 70% 27%

The United Motors contract uses only 3% of total machines-hours in 2007, yet uses
28% of CAD design-hours and 19% of engineering hours. The result is that the
plantwide rate, based on machine-hours, will greatly underestimate the cost of
resources used on the United Motors contract. Hence, the 157% increase in indirect
costs assigned to the United Motors contract when department rates are used.

In contrast, the Holden Motors contract uses less of design (51%) and engineering
(16%) than of machine-hours (70%). Hence, department rates will report lower
indirect costs than does a plantwide rate.

Page 46 CMA Ontario – January 2008


Module 3 - Management Accounting

Exercise 5-23

1. POR’s -
Machining: $450,000 / 75,000 = $6 per machine hour
Setup: $144,000 / 100 = $1,440 per production run
Inspection: $126,000 / 1,500 = $84 per inspection hour

Mathematical Financial
Machining: $6 x 25,000 | 50,000 $150,000 $300,000
Setup: $1,440 x 50 | 50 72,000 72,000
Inspections; $84 x 1,000 | 500 84,000 42,000
$306,000 $414,000
Divide by number of units 50,000 100,000
Overhead cost per unit $6.12 $4.14

2. Direct materials cost:


$180,000 / 50,000 | $360,000 / 100,000 $3.60 $3.60
Direct manufacturing labour
$60,000 / 50,000 | $120,000 / 100,000 1.20 1.20
Overhead cost per unit 6.12 4.14
$10.92 $8.94

Problem 5-30

1. Pricing decisions at Wigan Associates are heavily influenced by reported cost


numbers. Suppose Wigan is bidding against Hull & Kingston for a client with a job
similar to that of Widnes Coal. If the costing system overstates the costs of these
jobs, Wigan may bid too high and fail to land the client. If the costing system
understates the costs of these jobs, Wigan may bid low, land the client, and then
lose money in handling the case.

2. Not required.

3. Widnes Coal: 104 hours x ($84 + 126) = 104 hours x $210 = $21,840
St. Helen’s Glass: 96 hours x $210 = $20,160

Page 47 CMA Ontario – January 2008


Module 3 - Management Accounting

Problem 5-31

1. Not required.

2. $8,400 / 200 hours = $42.00 per hour

3. Widnes St’ Helen’s


Coasl Glass
Professional labour time: $84 x 104 | 96 $8,736 $8,064
Direct costs 3,780 13,020
Indirect cost allocation: $42 x 104 | 96 4,368 4,032
$16,884 $25,116

4. Cost using a single rate $21,840 $20,160


Cost using multiple direct cost categories $16,884 $25,116

The single rate approach directly traces to the individual jobs $16,800 that is
allocated in the multiple direct cost categories approach on the basis of direct
professional labour-hours. The averaging assumption implicit in the single rate
approach appears incorrect––for example, the St. Helen’s Glass job has travel costs
over seven times higher than the Widnes Coal case despite having lower direct
professional labour-hours.

Problem 5-32

1. Not required.

2. Widnes St’ Helen’s


Coasl Glass
Partner time: $120 x 24 | 56 $2,880 $6,720
Manager time: $60 x 80 | 40 4,800 2,400
Direct costs 3,780 13,020
Indirect cost allocation:
Partner time: $69 x 24 | 56 1,656 3,864
Manager time: $24 x 80 | 40 1,920 960
$15,036 $26,964

Page 48 CMA Ontario – January 2008


Module 3 - Management Accounting

3. The specific areas where the third approach can provide better information for
decisions at Wigan Associates include:

(Note the first approach refers to the single cost pool approach used in Problem 5-
30, the second approach refers to the separation of direct costs in Problem 5-31 and
the third approach refers to the separation of partner and manager time in this
problem)

a. Pricing and product (case) emphasis decisions. In a bidding situation using


data from the first approach, Wigan may win legal cases on which it will
subsequently lose money. It may also not win legal cases on which it would
make money with a lower-priced bid. The third approach signals to senior
managers those legal cases that are the most profitable; these signals can be
used in decisions where Wigan should increase its business development
efforts.

From a strategic viewpoint, the first approach exposes Wigan Associates to


cherry-picking by competitors. Other law firms may focus exclusively on
Widnes Coal-type cases and take sizable amounts of “profitable” business
from Wigan Associates. The third approach reduces the likelihood of Wigan
Associates losing cases on which it would have made money.

b. Client relationships. The third approach provides a better “road map” for
clients to understand how costs are accumulated at Wigan Associates. Wigan
can use this road map when meeting with clients to plan the work to be done
on a case before it commences. Clients can negotiate ways to get a lower-cost
case from Wigan, given the information in the third approach––for example,
(a) use a higher proportion of manager labour time and a lower proportion of
partner time, and (b) use fax machines more and air travel less. If clients are
informed in advance how costs will be accumulated, there is less likelihood of
disputes about bills submitted to them after the work is done.

c. Cost control. The third approach better highlights the individual cost areas at
Wigan Associates than does the first approach.

The third approach is more likely to promote better cost-control practices than
first approach (as the nine cost categories in the third approach may differ in
terms of how to effectively manage costs).

Page 49 CMA Ontario – January 2008


Module 3 - Management Accounting

Problem 5-39

1. General Mom/Pop
Super-mkt Drugstore Single
Chains Chains Stores Total
Revenues * $4,449,600 $3,780,000 $2,376,000 $10,605,600
Cost of goods sold ** 4,320,000 3,600,000 2,160,000 10,080,000
Gross margin $ 129,600 $ 180,000 $ 216,000 525,600
Operating costs 361,296
Operating income $ 164,304

Gross margin % 2.9% 4.8% 9.1%

* Average revenue per delivery x number of deliveries


** Average cost of goods sold per delivery x number of deliveries

2. Customer purchase order processing: $96,000 / 2,000 = $48 per order


Line item ordering: $76,608 / 21,280 = $3.60 per line item
Store delivery: $85,200 / 1,420 = $60 per delivery
Cartons shipped to stores: $91,200 / 76,000 = $1.20 per carton
Shelf-stocking at customer stores: $12,288 / 640 = $19.20 per hour

3. Allocation of Operating Costs - General Mom/Pop


Super-mkt Drugstore Single
Chains Chains Stores
Customer purchase order processing:
$48 x 140 | 360 | 1,500 $6,720 $17,280 $72,000
Line item ordering:
$3.60 x 1,960 | 4,320 | 15,000 7,056 15,552 54,000
Store delivery:
$60 x 120 | 300 | 1,000 7,200 18,000 60,000
Cartons shipped to stores:
$1.20 x 36,000 | 24,000 | 16,000 43,200 28,800 19,200
Shelf-stocking at customer stores:
$19.20 x 360 | 180 | 100 6,912 3,456 1,920
$71,088 $83,088 $207,120

Page 50 CMA Ontario – January 2008


Module 3 - Management Accounting

General Mom/Pop
Super-mkt Drugstore Single
Chains Chains Stores Total
Revenues $4,449,600 $3,780,000 $2,376,000 $10,605,600
Cost of goods sold 4,320,000 3,600,000 2,160,000 10,080,000
Gross margin 129,600 180,000 216,000 525,600
Operating costs 71,088 83,088 207,120 361,296
Operating income $58,512 96,912 $8,880 $ 164,304

Operating Income % 1.3% 2.6% 0.4%

The activity-based analysis of costs highlights how the "Mom and Pop" Single
Stores use a larger amount of Figure Four resources per revenue dollar than do the
other two markets.

4. • Choosing the appropriate cost drivers for each area. The case gives a cost
driver for each chosen activity area. However, it is likely that over time
further refinements in cost drivers would occur. For example, not all store
deliveries are equally easy to make, depending on parking availability,
accessibility of the storage/shelf space to the delivery point, etc. Similarly,
not all cartons are equally easy to deliver-their weight, size, or likely
breakage component are factors that can vary across carton types.
• Developing a reliable database on the chosen cost drivers. For some items,
such as the number of orders and the number of line items, this information
likely would be available in machine readable form at a high level of
accuracy. Unless the delivery personnel have hand-held computers that they
use in a systematic way, estimates of shelf-stocking time are likely to be
unreliable. Advances in information technology likely will reduce problems
in this area over time.
• Deciding how to handle costs that may be common across several activities.
For example, (3) store delivery and (4) cartons shipped to stores share the
common costs of the same trip. Some organizations may treat (3) as the
primary activity and attribute to (4) only incremental costs. Similarly, (1)
order processing and (2) line item ordering may have common costs.
• Choice of the time period to compute cost rates per cost driver. Flair
calculates driver rates on a monthly basis (August 2007). He may want to
consider using longer time periods that may be less affected by seasonal or
random variations in demand.
• Behavioural factors are likely to be a challenge to Flair. He must now tell
those salespeople who specialize in "Mom and Pop" accounts that they have
been less profitable than previously thought.

Page 51 CMA Ontario – January 2008


Module 3 - Management Accounting

7. Cost Allocations – Service Departments

Exercise 14-17

1. Rockford Peoria Hammond Kankakee


Practical capacity:
10,000 x $0.35* $3,500
20,000 x $0.35 $7,000
12,000 x $0.35 $4,200
8,000 x $0.35 $2,800

$17,500 / 50,000 = $0.35 per kilowatt-hour

Average expected usage:


8,000 x $0.5833* $4,667
9,000 x $0.5833 $5,250
7,000 x $0.5833 $4,083
6,000 x $0.5833 $3,500

$17,500 / 30,000 = $0.5833 per kilowatt-hour

2. Fixed cost rate: $10,500 / 50,000 = $0.21


Variable cost rate: $7,000 / 30,000 = $0.2333

Rockford Peoria Hammond Kankakee


Fixed costs
10,000 x $0.21 $2,100
20,000 x $0.21 $4,200
12,000 x $0.21 $2,520
8,000 x $0.21 $1,680

Variable costs
8,000 x $0.2333 1,867
9,000 x $0.2333 2,100
7,000 x $0.2333 1,633
6,000 x $0.2333 1,400
$3,967 $6,300 $4,153 $3,080

The dual-rate method permits a more refined allocation of the power department
costs; it permits the use of different allocation bases for different cost pools. The
fixed costs result from decisions most likely associated with the practical capacity
level. The variable costs result from decisions most likely associated with monthly
usage.

Page 52 CMA Ontario – January 2008


Module 3 - Management Accounting

Exercise 14-19

1. a. Rate - $590,000 / 250 trips = $2,360 per trip

Orange juice: 150 trips x $2,360 = $354,000


Grapefruit juice: 100 trips x $2,360 = $236,000

b. Rate - $660,000 / 300 trips = $2,200 per trip

Orange juice: 200 trips x $2,200 = $440,000


Grapefruit juice: 100 trips x $2,200 = $220,000

2. When budgeted costs/budgeted quantities are used, the Orange Juice Division knows
at the start of 2006 that it will be charged a rate of $2,360 per trip. This enables it to
make operating decisions knowing the rate it will have to pay for transportation. In
contrast, when actual costs/actual quantities are used, the Orange Juice Division must
wait until year-end to know its transportation charges.

The use of actual costs/actual quantities makes the costs allocated to one user a
function of the actual demand of other users. In 2006, the actual usage was 300
trips, which is 50 trips above the 250 trips budgeted. These extra trips are one
explanation for the actual cost rate being less than the budgeted rate. In 2006, the
Orange Juice Division would have had lower costs had the actual rate been used.
However, the reverse also will occur when there is lower use than budgeted by
other plants.

Page 53 CMA Ontario – January 2008


Module 3 - Management Accounting

Exercise 14-20

1. a. Fixed indirect rate: $215,000 / 250 trips = $860

Orange Juice Division:


Variable indirect costs: $1,500 x 200 $300,000
Fixed indirect costs: $860 x 150 129,000
$429,000

Grapefruit Juice Division:


Variable indirect costs: $1,500 x 100 $150,000
Fixed indirect costs: $860 x 100 86,000
$236,000

2. Orange Grapefruit
Juice Juice
Division Division
Single rate method 1 (bud rate x bud quantities) $354,000 $236,000
Single rate method 2 (act rate x act quantities) 440,000 220,000
Dual rate 429,000 236,000

The dual rate changes how the fixed indirect cost component is treated. By using
budgeted trips made, the Orange Juice Division is unaffected by changes from its
own budgeted usage or that of other divisions.

Page 54 CMA Ontario – January 2008


Module 3 - Management Accounting

Exercise 14-21

1. Corporate Consumer
Sales Sales
Administrative:
$73,200 x 42/70 | 28/70 $43,920 $29,280
Information systems:
$234,900 x 1,920/3,520 | 1,600/3,520 128,127 106,773
$172,047 $136,053

2. Corporate Consumer
IS Sales Sales
Overhead $234,900
Administrative:
$73,200 x 21/91 | 42/91 | 28/91 16,892 $33,785 $22,523
$251,792
Information systems:
$251,792 x 1,920/3,520 | 1,600/3,520 137,341 114,451
$171,126 $136,974

3. Information systems could have been ranked first. This would have changed the
allocations as follows:

Corporate Consumer
Admin Sales Sales
Overhead $73,200
Information systems:
$234,900 x 320/3,840 | 1,920/3,840
| 1,600/3,840 19,575 $117,450 $97,875
$92,775
Administrative:
$92,775 x 42/70 | 28/70 55,665 37,110
$173,115 $134,985

Note that the order in which we allocate service departments under the step method
is mostly arbitrary. The only method that provides accurate allocation is the
reciprocal method.

Page 55 CMA Ontario – January 2008


Module 3 - Management Accounting

Exercise 14-22

1. The reciprocal allocation method explicitly includes the mutual services provided
among all support departments. Interdepartmental relationships are fully
incorporated into the support department cost allocations.

2. A = 73,200 + 320/3,840 IS
IS = 234,900 + 21/91 A

A = 73,200 + 0.08333IS (1)


IS = 234,900 + 0.23077A (2)

Substituting equation (2) into equation (1):

A = 73,200 + 0.08333 (234,900 + 0.23077A)


A = 73,200 + 19,574 + 0.01923A
0.98077A = 92,774
A = $94,593

IS = 234,900 + 0.23077(94,593)
IS = $256,729

Corporate Consumer
Sales Sales
Administrative:
$94,593 x 42/91 | 28/91 $43,658 $29,106
Information systems:
$256,729 x 1,920/3,840 | 1,600/3,840 128,365 106,970
$172,023 $136,076

3. The reciprocal method is more accurate than the direct and step-down methods
when there are reciprocal relationships among support departments.

Page 56 CMA Ontario – January 2008


Module 3 - Management Accounting

Exercise 14-26

1. a. GOVT CORP
Admin/HR:
$620,000 x 40/75 | 35/75 $330,667 $289,333
Information systems:
$2,420,000 x 30/90 | 60/90 806,667 1,623,333
$1,137,334 $1,902,666

b. IS GOVT CORP
$2,420,000
Admin/HR:
$620,000 x 25% | 40% | 35% 155,000 $248,000 $217,000
$2,575,000
Information systems:
$2,575,000 x 30/90 | 60/90 858,333 1,716,667
$1,106,333 $1,933,667

c. Admin/HR GOVT CORP


$620,000
Information systems:
$2,420,000 x 10% | 30% | 60% 242,000 $726,000 $1,452,000
$862,000
Admin/HR:
$862,000 x 40/75 | 35/75 459,733 402,267
$1,185,733 $1,854,267

2. GOVT CORP
Direct method $1,137,334 $1,902,666
Step down (Admin/HR first) 1,106,333 1,933,667
Step down (Information Systems) 1,185,733 1,854,267

The direct method ignores any services to other support departments. The step-
down method partially recognizes support to other service departments. The
information systems support group (with total budget of $2,420,000) provides 10%
of its services to the A/HR group. The A/HR support group (with total budget of
$620,000) provides 25% of its services to the information systems support group.

Page 57 CMA Ontario – January 2008


Module 3 - Management Accounting

3. There are no objective criteria to determine the sequence for allocating support
departments using the step-down method. The textbook proposes three approached
that can be used (described below), but the application of each of these methods is
somewhat arbitrary.

(a) Allocate service departments on a ranking of the % of their total services


provided to other service departments.

1. Administrative/HR 25%
2. Information Systems 10%

(b) Allocate service departments on a ranking of the total dollar amount in the
service departments.

1. Information Systems $2,420,000


2. Administrative/HR $ 620,000

(c) Allocate service departments on a ranking of the dollar amounts of service


provided to other service departments

1. Information Systems
(0.10 × $2,420,000) = $242,000
2. Administrative/HR
(0.25 × $620,000) = $155,000

The choice Phoenix should make will depend on the terms of the government
contract and the corporate contracts. Assume the government contract is cost plus and
no terms specify the method of step-down allocation, and that Phoenix bids for
corporate contracts in a highly competitive environment. Then Phoenix should
choose the method that maximizes the costs charged to the government. This will
increase revenue from government contracts. At the same time by reducing overhead
charges on the competitive corporate contracts, Phoenix can sustain a profit at a
lower bid than otherwise would be possible. In this case by allocating the IS costs
first, Phoenix reduces total overhead that must be charged to corporate clients from
$1,933,667 to $1,854,267. A word of caution, however, this question only provides
information for a single quarter. If Phoenix' contracts extend over longer time periods
and provide no flexibility to Phoenix to opportunely switch from one step-down
method to another each quarter, then it would be unwise to base a decision on a single
data point.

Page 58 CMA Ontario – January 2008


Module 3 - Management Accounting

Exercise 14-27

1. A = 620,000 + 0.10IS (1)


IS = 2,420,000 + 0.25A (2)

Substituting equation (1) into equation (2):

IS = 2,420,000 + 0.25 (620,000 + 0.10IS)


IS = 2,420,000 + 155,000 + 0.025IS
0.975IS = 2,575,000
IS = 2,641,026

A = 620,000 + 0.10(2,641,026)
A = 884,103

GOVT CORP
Admin/HR:
$884,103 x 40% | 35% $ 353,641 $ 309,436
Information systems:
$2,641,026 x 30% | 60% 792,308 1,584,616
$1,145,949 $1,894,052

2. GOVT CORP
Direct $1,137,334 $1,902,666
Step Down (IS First) 1,106,333 1,933,667
Step Down (Admin/HR First) 1,185,733 1,854,267
Reciprocal 1,145,949 1,894,052

The four methods differ in the level of service department cost allocation across
service departments. The level of reciprocal service department is material.
Administrative/HR supplies 25% of its services to Information Systems. Information
Systems supplies 10% of its services to Administrative/HR The Information
Department has a budget of $2,420,000 which is more than 290.3% higher than
Administrative / HR.

The reciprocal method recognizes all the interactions and is thus the most accurate.

Page 59 CMA Ontario – January 2008


Module 3 - Management Accounting

Problem 14-30

1. Direct Method -
X Y
A: $102,000 x 250/400 | 150/400 $63,750 $38,250
B: $42,000 x 100/500 | 400/500 8,400 33,600
$72,150 $71,850

Step Method – A First


B X Y
$42,000
A: $102,000
x 100/500 | 250/500 | 150/500 20,400 $51,000 $30,600
$62,400
B: $62,400 x 100/500 | 400/500 12,480 49,920
$63,480 $80,520

Step Method – B First A X Y


$102,000
B: $42,000
x 500/1,000 | 100/1,000 | 400/1,000 21,000 $4,200 $16,800
$123,000
A: $123,000 x 250/400 | 150/400 78,875 46,125
$81,075 $62,925

Reciprocal Method -

A = $102,000 + 0.5B (1)


B = $42,000 + 0.2A (2)

Substituting equation (2) into equation (1):

A = $102,000 + 0.5(42,000 + 0.2A)


A = $102,000 + 21,000 + 0.1A
0.9A = $123,000
A = $136,667

B = 42,000 + 0.2(136,667)
B = 69,333

X Y
A: $136,667 x 250/500 | 150/500 $68,334 $41,000
B: $69,333 x 100/1,000 | 400/1,000 6,933 27,733
$75,267 $68,733

Page 60 CMA Ontario – January 2008


Module 3 - Management Accounting

2. At first glance, it appears that the total support cost will be $42 per unit of power
plus the material handling costs. If so, Manes would be better off by purchasing
from the power company. However, the decision should be influenced by the
effects of the interdependencies and the fixed costs. Note that the power needs
would be less if they were purchased from the outside:

Outside Power
Units Needs
X 100
Y 400
A (500 units less 20% of 500 units, because
there is no need to service the nonexistent power
department) 400
900

Total costs: 900 x $42 = $37,800

In contrast, the total costs that would be saved by not producing the power inside
would depend on the effects of the decision on various costs:

Avoidable costs of
1,000 units of power
produced inside
Variable indirect labour and indirect materials costs $12,000
Supervision 10,000
Materials handling: $72,000 x 20%* 14,400
Probable minimum cost savings $36,400

Possible additional savings:


• Can any supervision in materials handling be saved
because of overseeing less volume? Minimum
savings is probably zero; the maximum is probably
20% of $10,000 or $2,000.
• Is any amortization a truly variable, wear-and-tear
type of cost?

* the power department uses 20% of materials handling


department

In the short-run (at least until a capital investment in equipment is necessary) the
data suggest continuing to produce internally because the costs eliminated would
probably be less than the comparable purchase costs.

Page 61 CMA Ontario – January 2008


Module 3 - Management Accounting

Problem 14-40

1. The overhead cost charged to each division for use of the Waterloo facility is:

(Budgeted % use by division of Waterloo facility)


x (Budgeted overhead costs at Waterloo Facility)

If the ASD division understates its budgeted use of the Waterloo facility and all
other divisions provide unbiased estimates of their budgeted use, the ASD division
will have a lower budgeted % use factor for the Waterloo facility, and thus a lower
overhead cost charge. If all divisions understate their budgeted use, those
division(s) providing the greatest understatements will be those benefiting by their
understatement.

2. Alternative approaches Waterloo might take include:


a. Charging a division a penalty rate when it uses a higher number of hours than
it submitted as its budgeted amount.
b. Change the charge structure so that each hour of research scientist time used
has a budgeted overhead charge component. (This approach only partially
reduces the problem because the fixed overhead rate per hour must be
determined.)
c. Use actual costs per hour rather than budgeted costs per hour as the charge to
each using division

3. Goodwin’s first response should be to develop a well-constructed argument to


present to Roy Masters for using the 30,000 number. Masters should be given at least
one more chance to respond to Goodwin’s concerns. Ideally, Goodwin should give
Masters a short time period (say one week) to think about her concerns. This is
especially the case if there is already documentation at ASD for the 30,000 number.
Goodwin might note that if internal control people at WS are called in to consider
any allegations by other divisions that ASD is deliberately understating budgeted
usage, the 30,000 figure likely will be observable.

If Masters continues with his threats about dropping Goodwin from “the ASD
team,” she should contact the corporate controller at WS to seek guidance on how
to handle the situation.

Page 62 CMA Ontario – January 2008


Module 3 - Management Accounting

8. Joint and By-Product Costing

Exercise 15-16

1. a. Sales Value Allocation of


at Split-Off % Joint Costs
Breasts $132.00 67.5% $67.50
Wings 9.60 4.9% 4.90
Thighs 33.60 17.2% 17.20
Bones 19.20 9.8% 9.80
Feathers 1.20 0.6% 0.60
$195.60 $100.00

Cost of destroyed product:


Breasts: $67.50 / 100 kg x 30 kg $20.25
Wings: $4.90 / 20 kg x 15 3.68
$23.93

b. Physical
Measure Allocation of
Method % Joint Costs
Breasts 100 40% $40.00
Wings 20 8% 8.00
Thighs 40 16% 16.00
Bones 80 32% 32.00
Feathers 10 4% 4.00
250 $100.00

Cost of destroyed product:


Breasts: $40.00 / 100 kg x 30 kg $12.00
Wings: $8.00 / 20 kg x 15 6.00
$18.00

2. The sales-value at splitoff method captures the benefits-received criterion of cost


allocation. The costs of processing a chicken are allocated to products in proportion to
the ability to contribute revenue. Chicken Little’s decision to process chicken is heavily
influenced by the revenues from breasts and thighs. The bones provide relatively few
benefits to Chicken Little despite their high physical volume.

The physical measures method shows profits on breasts and thighs and losses on bones
and feathers. Given that Chicken Little has to jointly process all the chicken products, it is
counter-intuitive to single out individual products that are being processed
simultaneously as making losses while the overall operations make a profit.

Page 63 CMA Ontario – January 2008


Module 3 - Management Accounting

Problem 15-17

1. Breasts: $67.50 / 100 kg x 15 kg $10.13


Wings: $4.90 / 20 kg x 6 kg 1.47
Thighs: $17.20 / 40 kg x 4 kg 1.72
Bones: $9.80 / 80 kg x 9 kg 1.10
Feathers: $0.60 / 10 kg x 3 kg 0.18
$14.60

2. Joint costs to be allocated = $100 – 9.60 – 19.20 – 1.20 = $70.00

Sales Value Allocation of


at Split-Off % Joint Costs
Breasts $132.00 79.7% $55.79
Thighs 33.60 20.3% 14.21
$165.60 $70.00

Ending Inventory - Joint Products:


Breasts: $55.79 / 100 kg x 15 kg $8.37
Thighs: $14.21 / 40 kg x 4 kg 1.42
$9.79

Ending Inventory – By-Products:


Wings: 6 x $0.48 $2.88
Bones: 9 x $0.24 2.16
Feathers: 3 x $0.12 0.36
$5.40

3. Joint costs to be allocated = $100.00

Sales Value Allocation of


at Split-Off % Joint Costs
Breasts $132.00 79.7% $79.70
Thighs 33.60 20.3% 20.30
$165.60 $100.00

Ending Inventory – Joint Products:


Breasts: $79.70 / 100 kg x 15 kg $11.96
Thighs: $20.30 / 40 kg x 4 kg 2.03
$13.99

4. Treating all products as joint products does not require judgments as between joint and
byproducts. In contrast, the approach in requirement 3 results in inventory values being
shown for only two of the five products, while the approach in requirement 2 still
provides inventory values for both joint products and byproducts.

Page 64 CMA Ontario – January 2008


Module 3 - Management Accounting

Exercise 15-24

1. Incremental revenues:
Sales revenues from further processing:
10,000,000 x 0.95 x $43.20 $410,400,000
Sales revenue from bulk raw coal:
10,000,000 x $32.40 324,000,000
86,400,000
Incremental costs -
Direct labour $ 720,000
Supervisory personnel 120,000
Heavy equipment rental: $30,000 x 12 mo 360,000
Sizing and cleaning: 10,000,000 x $4.20 42,000,000
Freight: 10,000,000 x 0.95 / 60 x $288 45,600,000 88,800,000
Incremental loss ($2,400,000)

Newcastle Mining Company to continue to sell raw bulk coal without further
processing.

2. a. Coal files: 10,000,000 x 5% x 75% = 375,000 tonnes

Potential additional revenues


Minimum: 375,000 x $16.80 = $6,300,000
Maximum: 375,000 x $28.80 = $10,800,000

Since the incremental loss is $2.4 million, as calculated in requirement 1,


including the coal fines in the analysis indicates that further processing
provides a positive result and is, therefore, favourable.

b. Other factors that should be considered in evaluating a sell-or-process-further


decision include:
• Stability of the current customer market and how it compares to the
market for sized and cleaned coal.
• Storage space needed for the coal fines until they are sold and the
handling costs of coal fines
• Reliability of cost (e.g., rail freight rates) and revenue estimates, and the
risk of depending on these estimates
• Timing of the revenue stream from coal fines and impact on the need for
liquidity.
• Possible environmental problems, i.e., dumping of waste and dust from
unprocessed coal

Page 65 CMA Ontario – January 2008


Module 3 - Management Accounting

Exercise 15-27

1. a. Sales Value at Allocation of


Split-Off % Joint Costs
Chocolate Powder: 200 x $25.20 $5,040 35% $4,200
Milk Chocolate: 300 x $31.20 9,360 65% 7,800
$14,400 $12,000

b. Physical Allocation of
Measure % Joint Costs
Chocolate Powder 200 40% $4,800
Milk Chocolate 300 60% 7,200
500 $12,000

c. Net
Realizable Allocation of
Value % Joint Costs
Chocolate Powder
(2,000 x $4.80) – $5,100 $4,500 31.25% $3,750
Milk Chocolate
(3,400 x $6.00) - $10,500 9,900 68.75% 8,250
$14,400 $12,000

d. Sales value after further processing


(2,000 x $4.80) + (3,400 x $6.00) $30,000
Joint and separable costs
$12,000 + 5,100 + 10,500 27,600
$ 2,400
Gross profit % = $2,400 / 30,000 = 8%

Chocolate Milk
Powder Chocolate
Expected final sales value
2,000 x $4.80 | 3,400 x $6.00 $9,600 $20,400
Less gross margin at 8% (768) (1,632)
Less costs to process further (5,100) (10,500)
Joint costs allocated $3,732 8,268

Page 66 CMA Ontario – January 2008


Module 3 - Management Accounting

2. Chocolate Milk
Powder Chocolate
a. Sales Value at Split-off -

Sales $9,600 $20,400


Joint costs (4,200) (7,800)
Separable costs (5,100) (10,500)
Gross margin $300 $2,100

Gross margin % 3.1% 10.3%

b. Physical Measures

Sales $9,600 $20,400


Joint costs (4,800) (7,200)
Separable costs (5,100) (10,500)
Gross margin ($300) $2,700

Gross margin % (3.1%) 13.2%

c. Estimated NRV -

Sales $9,600 $20,400


Joint costs (3,750) (8,250)
Separable costs (5,100) (10,500)
Gross margin $750 $1,650

Gross margin % 7.8% 8.1%

d. 8% for both products

3. Chocolate Milk
Powder Chocolate
Revenues after processing further
(2,000 x $4.80) | (3,400 x $6.00) $9,600 $20,400
Revenues at split-off point
(200 x $25.20) | (300 x $31.20) 5,040 9,360
Incremental revenues 4,560 11,040
Costs of processing further 5,100 10,500
Incremental margin from processing further ($540) $540

Roundtree Chocolates could increase operating income by $540 (to $2,940) if


chocolate-powder liquor base is sold at the splitoff point and if milk-chocolate
liquor base is further processed into milk chocolate

Page 67 CMA Ontario – January 2008


Module 3 - Management Accounting

Problem 15-29

1. a. Sales Value at Allocation of


Split-Off % Joint Costs
Select oak $288,000 48% $172,800
While oak 240,000 40% 144,000
Knotty oak 72,000 12% 43,200
$600,000 $360,000

b. Physical Allocation of
Measure % Joint Costs
Select oak 30,000 30% $108,000
While oak 50,000 50% 180,000
Knotty oak 20,000 20% 72,000
100,000 $360,000

c. Net
Realizable % Allocation of
Value Joint Costs
Select oak* $408,000 48.6% $174,960
While oak** 324,000 38.6% 138,960
Knotty oak*** 108,000 12.8% 46,080
$840,000 $360,000

* (25,000 x $19.20) – 72,000


** (40,000 x $10.80) – 108,000
*** (15,000 x $8.40) – 18,000

2. a. Select White Knotty


Sales Value at Split-off -
Joint costs allocated $172,800 $144,000 $43,200
Separable processing costs 72,000 108,000 18,000
Total costs 244,800 252,000 61,200
Production volume 25,000 40,000 15,000
Cost per board-feet $9.79 $6.30 $4.08
Ending inventory in board feet 1,000 2,000 500
Ending inventory value $9,790 $12,600 $2,040

Page 68 CMA Ontario – January 2008


Module 3 - Management Accounting

b. Select White Knotty


Physical Measures -
Joint costs allocated $108,000 $180,000 $72,000
Separable processing costs 72,000 108,000 18,000
Total costs 180,000 288,000 90,000
Production volume 25,000 40,000 15,000
Cost per board-feet $7.20 $7.20 $6.00
Ending inventory in board feet 1,000 2,000 500
Ending inventory value $7,200 $14,400 $3,000

c. Select White Knotty


Estimated NRV -
Joint costs allocated $174,960 $138,960 $46,080
Separable processing costs 72,000 108,000 18,000
Total costs 246,960 246,960 64,080
Production volume 25,000 40,000 15,000
Cost per board-feet $9.878 $6.174 $4.272
Ending inventory in board feet 1,000 2,000 500
Ending inventory value $9,878 $12,348 $2,136

Select White Knotty


3. Revenues after processing further
25,000 x $19.20 $480,000
40,000 x $10.80 $432,000
15,000 x $8.40 $126,000
Revenues at split-off point
30,000 x $9.60 288,000
50,000 x $4.80 240,000
20,000 x $3.60 72,000
Incremental revenues 192,000 192,000 54,000
Costs of processing further 72,000 108,000 18,000
Incremental margin from processing
further $120,000 $84,000 $36,000

Pacific Lumber is maximizing its total August 2007 operating income by fully
processing each raw oak product into its finished product form.

Page 69 CMA Ontario – January 2008


Module 3 - Management Accounting

Problem 15-33

1. Net realizable value of Alpha:


Sales value of Alpha: 46,200 kg x $6 $277,200
Sales value of Beta: 19,800 kg x $1.44 28,512
Separable processing costs: $28,392 + 9,720 + 45,600 (83,712)
$222,000

Net realizable value of Gamma:


Sales value of Gamma : 40,000* x $14.40 $576,000
Separable processing costs (198,000)
$378,000

Net
Realizable % Allocation of
Value Joint Costs
Alpha $222,000 37% $53,280
Gamma 378,000 63% 90,720
$600,000 $144,000

2. Sales (46,200 kg x 80% x $6) $221,760


Cost of goods sold -
Allocated joint costs $53,280
Separable costs ($45,600 + 28,392) 73,992
NRV of Beta [(19,800 kg x $1.44) - $9,720] (18,792)
108,480
Less ending inventory ($108,480 x 20%) (22,696) 86,784
Gross margin $134,976

* good output = 44,000 / 1.1 = 40,000

Page 70 CMA Ontario – January 2008


Module 3 - Management Accounting

9. In-Class Problems

Problem 1 – Cost Classifications

The following information appears in Marshall Inc.’s records for the year ended
December 31, 20x6:

Sales revenue $131,150


Work in process, January 1 7,700
Work in process, December 31 6,210
Direct materials inventory, January 1 8,600
Direct materials inventory, December 31 8,050
Finished goods inventory, January 1 3,550
Finished goods inventory, December 31 4,950
Direct materials transportation-in 1,150
Direct materials purchased 11,560
Direct labor 19,350
Supervisory and indirect labor-plant 10,950
Administrative salaries 18,000
Supplies and indirect materials-plant 1,450
Heat, light, and power (77.6% for plant) 12,500
Depreciation (80% for plant) 15,000
Property taxes (75% for plant) 4,200
Other administrative costs 4,350
Marketing costs 16,350

Prepare an income statement with a supporting cost of goods manufactured statement.

Page 71 CMA Ontario – January 2008


Module 3 - Management Accounting

Problem 2 – Job Order Costing

Prepare journal entries to record the following transactions:

(a) Purchased $40,000 of raw materials on account


(b) Issued $25,000 of direct materials to jobs
(c) Incurred direct labour costs of $50,000
(d) Paid $53,000 cash for factory utilities
(e) Incurred indirect labour costs of $18,000
(f) Issued $3,000 of indirect materials to the factory floor
(g) Applied overhead on the basis of 125% of direct labour cost
(h) Incurred depreciation charges on factory machinery of $10,000
(i) The cost of jobs completed was $60,000
(j) The cost of jobs sold was $56,000

Page 72 CMA Ontario – January 2008


Module 3 - Management Accounting

Problem 3 – Job Order Costing

The Adams Company has two jobs in work-in-process at the beginning of the January:

Total
Cost
Job 1006 $18,200
Job 1007 13,500

During the January, three additional jobs were started: Jobs 1008, 1009 and 1010. During
the month, jobs 1006, 1007 and 1008 were completed and Jobs 1006 and 1007 were sold.
There were no finished goods inventory at the beginning of the month.

The following transactions took place during the month:

a. Direct materials used and direct labour hours incurred in January by each job
were as follows:

1006 1007 1008 1009 1010


Direct materials used $1,200 $2,250 $4,600 $5,700 $1,200
Direct labour hours 20 35 86 140 25

b. The company applies overhead on the basis of direct labour hours. At the
beginning of the year, it was estimated that the total overhead costs for the year
would be $75,000 and that total direct labour hours incurred would be 3,000.

c. The direct labour rate is $30 per hour.

d. Raw materials inventory at the beginning of January was $5,000, purchases of


raw materials during January amounted to $16,300.

e. The following overhead costs were incurred during the month:

Indirect materials used $ 350


Indirect labour 1,600
Utilities 1,200
Depreciation 2,100
Other 500
$5,750

Required –

1. Prepare all journal entries for the month of January.


2. Post the journal entries to the WIP Account and show that the ending balance in
the ending WIP account corresponds to the cost of the remaining jobs.

Page 73 CMA Ontario – January 2008


Module 3 - Management Accounting

Problem 4 – Job Order Costing

Avid Assemblers uses job-order costing to assign costs to products. The company
assembles and packages 20 different products to customer specifications. Products are
worked on in batches of 30 to 50 units. Each batch is assigned a job number.

On October 1, the company had the following balances recorded.

Direct Materials $ 7,800


Work in process 45,726
Finished goods 23,520

Work in process consisted of the following jobs:

Job 202 Job 204 Job 205


Direct materials $ 4,200 $ 3,190 $ 2,800
Direct labour 8,500 7,210 6,500
Applied overhead 5,100 4,326 3,900
Total $17,800 $14,726 $13,200
Number of units 30 50 35

Finished goods consisted of job 203, with the following costs:

Direct materials $ 7,200


Direct labour 10,200
Applied overhead 6,120
Total $23,520
Number of units 50

Shown below are the direct cost data related to jobs started in October:

Job 206 Job 207 Job 208 Total


Direct materials $4,180 $3,600 $1,200 $ 8,980
Direct labour 9,200 8,340 2,910 20,450
Number of units 40 50 40

Page 74 CMA Ontario – January 2008


Module 3 - Management Accounting

Other information:

1. Direct materials and direct labour added to beginning work in process in October
were as follows:

Job 202 Job 204 Job 205 Total


Direct materials $ 950 $ 410 $1,200 $ 2,560
Direct labour 2,000 3,500 4,500 10,000

2. Overhead is applied at a predetermined rate on the basis of direct labour cost.

3. Actual expenses for October were as follows:

Supervisory salaries $4,000


Factory rent 2,000
Amortization (machines) 3,000
Indirect labour 5,000
Supplies (factory) 1,100
Selling expenses 8,500
CPP, El, and other benefits* 3,200

*80 percent of employer contributions and benefits relates to factory personnel.

4. Purchases of direct materials during October amounted to $8,500

5. Only Jobs 207 and 208 are still in process at closing on October 31. Finished
goods consisted only of Job 205 at month end.

6. Avid writes off any over- or underapplied overhead to Cost of Goods Sold in the
month it is incurred.

Required:

1. What is the predetermined overhead rate used by Avid to apply overhead to jobs?
2. What is the unit cost of Job 204 in October?
3. What are the October 31 balances for the following inventory accounts?
a. Direct Materials
b. Work in Process
c. Finished Goods
4. What is the cost of goods manufactured in October? (You do not have to prepare
a statement as part of this Requirement.)
5. Determine the over- or underapplied overhead for October.

Page 75 CMA Ontario – January 2008


Module 3 - Management Accounting

Problem 5 – Process Costing

Materials are added at the beginning of production for the Jordan Company, and
overhead is applied to each product at the rate of 60% of direct labour costs. Labour and
overhead are incurred uniformly throughout the process. A review of Jordan's inventory
cost records provides you with the following information for the month of July:

Direct Direct
Materials Labour
Units Costs Costs
Work-in-process, beginning of July
(80% complete as to labor and overhead) 200,000 $ 200,000 $ 315,000
Units started in production 1,000,000
Units completed in July 900,000
Costs for July 1,300,000 1,995,000

Ending work in process is 50% complete as to labour and overhead.

Required:

Calculate the cost of the units transferred out to finished goods inventory and the value of
the ending WIP under both the FIFO method and the weighted average method.

Page 76 CMA Ontario – January 2008


Module 3 - Management Accounting

Problem 6 – Process Costing

Deterra, Inc., uses three departments to produce a detergent. The Finishing Department is
the third and last step before the product is transferred to storage. All materials needed to
give the detergent its final composition are added at the beginning of the process in the
Finishing Department. The company uses FIFO costing. The following data for the
Finishing Department for October have been made available:

Production data:

In process, October 1 (labor and factory overhead, 75% complete) 10,000 litres
Transferred in from preceding department 40,000 litres
Finished and transferred to storage 35,000 litres
In process, October 31 (labor and factory overhead, 50% complete) 15,000 litres

Additional data:

Work-in-process inventory, October 1:


Cost from preceding department $ 38,000
Cost from this department:
Materials 21,500
Labor 39,000
Factory overhead 42,000
$140,500

Transferred in during October $140,000

Cost added in this department:


Materials $ 70,000
Labor 162,500
Factory overhead 130,000
$362,500

Required:

Calculate the cost of the units transferred out to finished goods inventory and the value of
the ending WIP under both the FIFO method and the weighted average method.

Page 77 CMA Ontario – January 2008


Module 3 - Management Accounting

Problem 7 – Job Order Costing

Jessica Company started operations on January 2, 20x6. The company manufactures


custom products and uses a job order system. Overhead is allocated to jobs based on
direct labour costs. The budgeted manufacturing overhead for 20x6 was $396,900 and the
direct labour costs were budgeted at $567,000. At the end of 20x6, there were two jobs in
work in process:

Job A605 Job A608


Direct material cost $20,000 $36,000
Direct labour cost 12,000 38,000

Actual manufacturing overhead for the year amounted to $350,000 and total direct labour
charges for the year amounted to $550,000. The year-end finished good inventory
balance was $175,000 and included direct labour costs of $48,000. Cost of goods sold for
the year amounted to $1,750,000.

Required -

1. Prepare a schedule showing the detailed cost of the ending work-in-process, and
finished goods inventory.
2. Compute the over/under –applied overhead for the year.
4. Allocate the balance in the manufacturing overhead account using each of the 4
approaches discussed in class.

Page 78 CMA Ontario – January 2008


Module 3 - Management Accounting

Problem 8 – Process Costing with Spoilage

Kristina Company, which manufactures quality paint sold at premium prices, uses a
single production department. Production begins with the blending of various chemicals,
which are added at the beginning of the process, and ends with the canning of the paint.
Canning occurs when the mixture reaches the 90% stage of completion. The gallon cans
are then transferred to the Shipping Department for crating and shipment. Labor and
overhead are added continuously throughout the process.

Prior to May, when a change in the process was implemented, WIP inventories were
insignificant. The change in the process enables greater production but results in material
amounts of WIP for the first time. The company has always used the WA method to
determine equivalent production and unit costs. Now, production management is
considering changing from the WA method to the FIFO method.

Spoilage is detected when the mixture reaches the 60% stage of completion. Normal
spoilage is equal to 2% of units transferred out.

The following data relate to actual production during the month of May:

COSTS FOR MAY


Work-in-process inventory, May 1
(4,000 litres 25% complete):
Direct materials-chemicals $ 45,600
Direct labor 6,250
Factory overhead 18,750

May costs added:


Direct materials-chemicals 228,400
Direct materials-cans 7,000
Direct labor 35,000
Factory overhead 105,000

UNITS FOR MAY - Litres


Work-in-process inventory, May 1 (25% complete) 4,000
Sent to Shipping Department 19,350
Started in May 21,000
Work-in-process inventory, May 31 (80% complete) 5,000

Required –

Calculate the cost of units transferred out and the cost of abnormal spoilage using (a)
FIFO and (b) weighted average.

Page 79 CMA Ontario – January 2008


Module 3 - Management Accounting

Problem 9 – Job Order Costing and Spoilage

The Dangelo Company manufactures products that often require specification changes or
modifications to meet its customers' needs. Still, Dangelo has been able to establish a
normal spoilage rate of 2.5% of normal input. Normal spoilage is recognized during the
budgeting process and classified as a component of manufacturing overhead when
determining the overhead rate.

One of Dangelo's inspection managers, obtains the following information for Job No.
A604 that was recently completed. A total of 122,000 units were started, and 5,000 units
were rejected at final inspection yielding 117,000 good units. Nine hundred of the first
units produced were rejected because of a design defect that was considered very
unusual; this defect was corrected immediately, and no further units were rejected for this
reason. These units were disposed of after incurring an additional cost of $1,200. The
inspection department was unable to identify a rejection pattern for the remaining 4,100
rejected units. These units can be sold at $7 per unit.

The total costs for all 122,000 units of Job No. A604 are presented here. The job has been
completed, but the costs have yet to be transferred to finished goods.

Direct materials $2,196,000


Direct manufacturing labour 1,830,000
Manufacturing overhead 2,928,000
Total manufacturing costs $6,954,000

Required -

1. Calculate the unit quantities of normal and abnormal spoilage.


2. Prepare the appropriate journal entry (or entries) to properly account for job No.
A604 including spoilage, disposal, and transfer of costs to finished goods control.

Page 80 CMA Ontario – January 2008


Module 3 - Management Accounting

Problem 10 – Cost Estimation

The following selected data were taken from the accounting records of Daviault
Manufacturing Company. The company uses direct-labor hours as its cost driver for
overhead costs.

Direct-Labor Manufacturing
Month Hours Overhead
January 26,000 $749,250
February 25,000 720,000
March 28,000 772,500
April 23,000 681,000
May 30,000 775,500
June 34,000 879,000

June's cost consisted of machine supplies ($153,000), depreciation ($22,500), and plant
maintenance ($703,500). These costs exhibit the following respective behavior: variable,
fixed, and semivariable (mixed).

The manufacturing overhead figures presented in the preceding table do not include
supervisory labor cost, which is step-fixed in nature. For volume levels of less than
15,000 hours, supervisory labor amounts to $67,500. The cost is $135,000 from 15,000-
29,999 hours and $202,500 when activity reaches 30,000 hours or more.

Required:

1. Determine the machine supplies cost and depreciation for April.


2. Using the high-low method, analyze Daviault Manufacturing Company's plant
maintenance cost and calculate the monthly fixed portion and the variable cost per
direct-labor hour.
3. Assume that present cost behavior patterns continue into the latter half of the year.
Estimate the total amount of manufacturing overhead the company can expect in
October if 29,500 direct-labor hours are worked.

Page 81 CMA Ontario – January 2008


Module 3 - Management Accounting

Problem 11 – Activity Based Costing

Mona Loa, Inc. (MLI), is a distributor and processor of a variety of different brands of
coffee. The company buys coffee beans from around the world and roasts, blends, and
packages them for resale. MLI currently has 10 different coffees that it offers to gourmet
shops in one kilogram bags. The major cost is direct materials, however, there is a
substantial amount of factory overhead in the predominantly automated roasting and
packing process. The company uses relatively little direct labor.

Some of the coffees are very popular and sell in large volumes, while a few of the newer
brands have very low volumes. MLI prices its coffee at full product cost, including
allocated overhead, plus a markup of 25 percent. If prices for certain coffees are
significantly higher than the market, the prices are lowered. The company competes
primarily on the quality of its products, but customers are price conscious as well.

Data for the 20x8 budget include factory overhead of $3,500,000, which has been
allocated in its current costing system on the basis of each product's direct labor cost. The
budgeted direct labor cost for 20x8 totals $700,000. Budgeted purchases and use of direct
materials (mostly coffee beans) will total $6,000,000.

Budgeted direct costs for one kg. bags of two of the company's products are as follows:

Italian French
Roast Roast
Direct materials $3.20 $2.80
Direct labor 0.30 0.30

MLI's controller believes the current traditional product costing system may be providing
misleading cost information. He has developed this analysis of the 20x8 budgeted factory
overhead costs:

Budgeted Budgeted
Activity Cost Driver Activity Cost
Purchasing Purchase order 1,150 $575,000
Material handling Setups 1,750 612,500
Quality control Batches 500 150,000
Roasting Roasting-hours 100,000 950,000
Blending Blending-hours 23,125 462,500
Packaging Packaging-hours 30,000 750,000
$3,500,000

Page 82 CMA Ontario – January 2008


Module 3 - Management Accounting

Data regarding the 20x8 production of Italian Roast and French Roast coffee follow.
Assume no beginning or ending direct materials inventory for either of these coffees.

Italian Roast French Roast


Budgeted sales 100,000 kg. 2,000 kg.
Batch size 10,000 kg. 500 kg.
Setups 3 per batch 3 per batch
Purchase order size 25,000 kg. 500 kg.
Roasting time 1 hour per 100 kg. 1 hour per 100 kg.
Blending time .5 hours per 100 kg. .5 hours per 100 kg.
Packaging time .1 hours per 100 kg. .1 hours per 100 kg.

Required –

1) Using MLI’s current traditional product costing system:


a) Determine the company’s predetermined overhead rate using direct labour cost as
the single cost driver.
b) Determine the full product costs and selling prices for one kg. of Italian Roast and
French Roast coffee.
2) Develop a new product cost, using an activity-based costing approach, for one kg. of
Italian Roast and French Roast coffee. Compare the results with those in requirement
1.

Page 83 CMA Ontario – January 2008


Module 3 - Management Accounting

Problem 12 – Process Costing

For each of the following independent cases, determine the information requested.

a. Beginning inventory amounted to 500 units. This period 2,250 units were started
and completed. At the end of the period, the 1,500 units in inventory were 30
percent complete. Using FIFO costing, the equivalent production for the period was
2,800 units. What was the percentage of completion of the beginning inventory?

b. The ending inventory included $8,700 for conversion costs. During the period,
4,200 equivalent units were required to complete the beginning inventory, and
6,000 units were started and completed. The ending inventory represented 1,000
equivalent units of work this period. FIFO costing is used. What were the total
conversion costs incurred this period?

c. In the beginning inventory, 1,000 units were 40 percent complete with respect to
materials. During the period, 8,000 units were transferred out. Ending inventory
consisted of 1,400 units that were 70 percent complete with respect to materials.
How many units were started and completed during the period? Assume FIFO
costing.

d. At the start of the period, 8,000 units were in the work in process inventory and
6,000 units were in the ending inventory. During the period, 19,000 units were
transferred out to the next department. Materials and conversion costs are added
evenly throughout the production process. FIFO costing is used. How many units
were started this period?

e. In the beginning inventory 4,100 units were 40 percent complete with respect to
conversion costs. During the period, 3,500 units were started. In the ending
inventory, 3,250 units were 20 percent complete with respect to conversion costs.
How many units were transferred out? Weighted average costing is used.

f. The beginning inventory consisted of 4,000 units with a direct materials cost of
$14,200. The equivalent work represented by all of the direct materials costs in the
WIP Inventory account amounted to 18,000 units. There were 6,000 units in ending
inventory that were 20 percent complete with respect to materials. The ending
inventory had a direct materials cost assigned of $4,500. What was the total
materials cost incurred this period? Weighted average costing is used.

g. The WIP Inventory account had a beginning balance of $1,900 for conversion costs
on items in process and during the period $18,100 in conversion costs were charged
to it. Also during the period, $19,200 in costs were transferred out. There were 400
units in the beginning inventory, and 4,800 units were transferred out during the
period. How many equivalent units are in the ending inventory? Weighted average
costing is used.

Page 84 CMA Ontario – January 2008


Module 3 - Management Accounting

h. During the period, 1,050 units were transferred to the department. The 1,600 units
transferred out were charged to the next department at an amount that included
$3,360 for direct materials costs. The ending inventory was 25 percent complete
with respect to direct materials and had a cost of $630 assigned to it. How many
units are in the ending inventory? Weighted average costing is used.

Problem 13 – Cost Allocations

Bio Labs is a genetic engineering firm manufacturing a variety of gene-spliced,


agricultural-based seed products. The firm has five separate laboratories producing
different product lines. Each lab is treated as a profit center and all five labs are located in
the same facility. The wheat seed lab and corn seed lab manufacture two of the five
product lines. These two labs are located next to each other and are of roughly equal size
in terms of sales. The two departments have close interaction, often sharing equipment
and lab technicians. Both use very similar technology and science and usually attend the
same scientific meetings.

Recent discoveries have shown how low-power lasers can be used to significantly
improve product quality. The wheat seed and corn seed managers are proposing the
creation of a laser testing department to employ this new technology. Leasing the
equipment and hiring the personnel cost $350,000 per year.

Supplies, power, and other variable costs are $25 per testing hour. The testing department
is expected to provide 2,000 testing hours per year. Wheat seed expects to use 700 testing
hours per year of the laser testing department and corn seed expects to use 800 testing
hours. The remaining 500 hours of testing capacity can be used by the other three labs if
the technology applies or can be left idle for future expected growth of the two
departments. Initially, only wheat and corn are expected to use laser testing.

The executive committee of Bio Labs has approved the proposal but is now grappling
with how to treat the costs of the laser testing department. The committee wants to charge
the costs to wheat seed and corn seed but is unsure of how to proceed.

At the end of the first year of operating the laser, wheat seed used 650 testing hours, corn
seed used 900 hours, and 450 hours were idle.

Required

a. Design two alternative cost allocation systems.


b. Give numerical illustrations of the charges the corn and wheat seed labs will incur
in the first year of operations under your two alternatives.

Page 85 CMA Ontario – January 2008


Module 3 - Management Accounting

Problem 14 – Joint and By Products

Willis Company produces three products: alpha, beta, and gamma. Alpha and gamma are
main products; beta is a by-product of alpha. Information on the past month's production
processes are as follows:

In Department 1, 110,000 units of the raw material X100 are processed at a total cost of
$290,000. After processing in Department 1, 60 percent of the units are transferred to
Department 2, and 40 percent of the units (now unprocessed gamma) are transferred to
Department 3.

In Department 2, the materials received from Department 1 are processed at an additional


cost of $76,000. Seventy percent of the units become alpha and are transferred to
Department 4. The remaining 30 percent emerge as beta and are sold at $4.20 per unit.
The additional processing costs to make beta salable are $16,200.

In Department 3, gamma is processed at an additional cost of $330,000. A normal loss of


units of gamma occurs in this department. The loss equals 10 percent of the units of good
output. The remaining good output is then sold for $24 per unit.

In Department 4, alpha is processed at an additional cost of $32,960. After this


processing, alpha can be sold for $10 per unit.

Required –

Prepare a schedule showing the allocation of the $290,000 joint cost between alpha and
gamma using the estimated net realizable value approach. Revenue from the sale of
byproducts should be credited to the manufacturing costs of the related main product.

Page 86 CMA Ontario – January 2008


Module 3 - Management Accounting

10. Multiple-Choice Questions

Week 14
Multiple Choice Questions

1. Maritime Company manufactures and sells Widget Q. For the year 20x3, the cost of
goods sold of Maritime Company was $350,000. During the year, the work-in-
process inventory increased by $42,000 and the finished goods inventory decreased
by $95,000. The total manufacturing costs incurred by Maritime Company in 20x3
were
a) $213,000
b) $297,000
c) $350,000
d) $403,000
e) $487,000

The following information pertains to questions 2-3:

Solids Company uses a normal cost accounting system. Budgeted costs for the year
were $4 per kg. of raw materials, $ 15 per direct labor hour and $20 per machine
hour. Budgeted production activity was 20,000 machine hours for the year. During
the year, the following were actually incurred for manufacturing:

Kg. of raw materials used 65,000


Direct labor hours incurred 15,000
Machine hours incurred 19,000
Raw materials costs $200,000
Direct labor costs $247,000
Factory overhead costs $358,000

2. How much was overhead overapplied or underapplied during the year?


a) $22,000 underapplied.
b) $20,000 underapplied.
c) $20,000 overapplied.
d) $22,000 overapplied.
e) $42,000 overapplied.

3. For a job that used 250 kgs. of direct materials, 45 direct labor hours and 50
machine hours, what was the cost of the job under normal costing?
a) $2,452.
b) $2,510.
c) $2,617.

Page 87 CMA Ontario – January 2008


Module 3 - Management Accounting

d) $2,675.
e) $2,683

The following information pertains to questions 4-5:

Markham Company uses a normal job order costing system. Overhead rates are
applied on the basis of machine hours in Department 1 and direct labor dollars in
Department 2. Budgeted data for the two departments are as follows:

Department 1 Department 2
Budgeted overhead $150,000 $300,000
Budgeted activity:
Direct labor hours (DLH) 50,000 100,000
Machine hours (MH) 30,000 50,000
Direct labor wage rate $20 per DLH $30 per DLH

4. The following data pertains to Job 94-669 which was completed during the year:

Direct materials $350,000


Direct labor hours (DLH):
- Department 1 1,000
- Department 2 2,000
Machine hours (MH):
- Department 1 5,000
- Department 2 2,000

The total factory overhead applied to Job 94-669 is


a) $ 9,000
b) $15,000
c) $31,000
d) $37,000
e) $80,000

5. During the year, actual direct labor wage rates and factory overhead incurred in
Departments 1 and 2 were as budgeted. Also, 52,000 direct labor hours and 35,000
machine hours were used in Department 1 for production, whereas 98,000 DLH and
46,000 MH were used in Department 2. Overhead for Markham Company is
a) $19,000 underapplied
b) $ 1,000 underapplied
c) $ 1,000 overapplied.
d) $19,000 overapplied
e) $31,000 overapplied.

Page 88 CMA Ontario – January 2008


Module 3 - Management Accounting

6. Zoom Corporation uses standard absorption costing based on machine hours to


account for overhead costs. The company bases its standards on estimated annual
sales of $5,000,000, annual overhead costs of $1,300,000 and annual machine hour
usage of 500,000 hours. During 20x4, however, Zoom Corporation experienced
$5,800,000 in sales and used 600,000 machine hours, and actual overhead costs
amounted to $1,400,000. For 20x4, overhead is
a) $108,000 overapplied
b) $100,000 underapplied
c) $160,000 overapplied
d) $260,000 overapplied
e) $208,000 overapplied

The following information pertains to items 7 – 9:

On January 31, a snow storm damaged the office of a small business and some of
the accounting information stored in the computer's memory was lost. The
following information pertaining to January activities was retrieved from other
sources:

Direct materials purchased $ 18,000


Work-in-process - beginning inventory $2,000
Direct materials - beginning inventory $6,000
Direct materials - ending inventory $10,000
Finished goods - beginning inventory $12,000
Finished goods - ending inventory $2,500
Sales $60,000
Manufacturing overhead and direct labor incurred $22,000
Gross profit percentage based on net sales 40%

7. The cost of direct materials used in January was


a) $8,000
b) $14,000
c) $18,000
d) $22,000
e) $28,000

8. Assume that $20,000 of direct materials were used in January. What amount of
work-in process was transferred out to finished goods during January?
a) $24,000
b) $26,500
c) $36,000
d) $38,000
e) $38,500

Page 89 CMA Ontario – January 2008


Module 3 - Management Accounting

9. Assume that $20,000 of direct materials were used in January and that cost of
goods available for sale in January amounted to $40,000. The ending work-in-
process inventory amounted to
a) $0
b) $4,000
c) $5,000
d) $14,000
e) $16,000

The following information pertains to question 10 -12:

Deejay Company uses an actual process costing system. In Department 2, direct


materials are added at the 50% stage of completion of the process and conversion
costs are added uniformly throughout the process. On March 1, there were 6,000
units in work-in-process and they were 60% completed as to conversion cost.
During March, 42,000 units were transferred into Department 2 for processing. On
March 31, there were 8,000 units in work-in-process: 3,000 units were 40%
complete as to conversion costs and the other 5,000 units were 80 percent complete
as to conversion costs. No spoilages were reported during March.

10. Using the FIFO method, the number of equivalent units of production for the
transferred-in costs would be
a) 8,000 higher than that of the weighted-average method.
b) 6,000 higher than that of the weighted-average method.
c) the same as that of the weighted-average method.
d) 6,000 lower than that of the weighted-average method
e) 8,000 lower than that of the weighted-average method.

11. Using the FIFO method, the number of equivalent units of production for direct
materials for March would be
a) 39,000
b) 40,000
c) 42,000
d) 45,000
e) 48,000

12. Using the weighted-average method, the number of equivalent units of production
for conversion costs would be
a) 39,200
b) 40,000
c) 41,600
d) 44,000
e) 45,200

Page 90 CMA Ontario – January 2008


Module 3 - Management Accounting

13. Which of the following is most likely NOT a discretionary cost?


a) Advertising.
b) Conferences.
c) Insurance.
d) Management consulting.
e) Executive training.

The following information pertains to question 14 – 15:

HTC Inc. is a manufacturing organization that uses actual absorption costing for
inventory costing. The following represents various HTC Inc. Year 4 account balances (in
.000s):

Property taxes on plant $ 40


Direct materials used 1,000
Direct labour used 200
Indirect labour used 120
Sales commissions 220
Plant electricity and water 180
Plant amortization 110
Indirect materials used 100
Plant repairs and maintenance 140
Finished goods inventory - Dec. 31, Year 3 230
Finished goods inventory - Dec. 31, Year 4 250
Work-in-process inventory - Dec. 31, Year 3 170
Work-in-process inventory - Dec. 31, Year 4 130

14. For financial reporting purposes, what amount of indirect manufacturing costs were
transferred to work in process during Year 4 (in .000s)?
a) $580
b) $910
c) $540
d) $220
e) $690

Page 91 CMA Ontario – January 2008


Module 3 - Management Accounting

15. Assume that total indirect manufacturing costs transferred to work in process
during Year 4 was $565 thousand. For financial reporting purposes, what was HTC
Inc.’s cost of goods sold for Year 4 (in .000s)?
a) $1,785
b) $2,005
c) $1,745
d) $1,625
e) $1,805

16. SME Ltd. manufactures electronic components through two processes: fabrication
and assembly. It has been determined that the appropriate cost driver to use for
allocating overhead costs is direct machine hours for the fabrication process and
direct labour hours for the assembly process. Budgeted overhead costs and activity
for the upcoming year are as follows

Fabrication Assembly
Total overhead costs $300,000 $180,000
Direct machine hours 10,000 3,000
Direct labour hours 8,000 9,000

The actual production data for Job 111, which was completed during the year, were
as follows:

Fabrication Assembly
Direct machine hours 10 50
Direct labour hours 40 25

What amount of overhead costs would have been applied to Job 111?

a) $3,300
b) $2,000
c) $4,500
d) $2,200
e) $800

Page 92 CMA Ontario – January 2008


Module 3 - Management Accounting

The following apply to questions 17 – 19:

Marian Manufacturing produces a product that passes through two departments. The units
from the Molding Department are completed in the Assembly Department. The units are
completed in Assembly by adding the remaining direct materials when the units are 60
percent complete with respect to conversion costs. Conversion costs are added
proportionately in Assembly. The production activity in the Assembly Department for the
current month is presented below. Marlan uses the FIFO (first-in, first-out) inventory
method in its process cost system. The ending work in process inventory was 40%
complete with regards to conversion costs.

Beginning inventory units


(25% complete with respect to conversion costs) 8,000
Units transferred in from the Molding Department during the month 42,000
Units completed and transferred to finished goods inventory 38,000

17. The equivalent units transferred from the Molding Department to the Assembly
Department for the current month would be
a) 30,000 units.
b) 38,000 units.
c) 40,800 units.
d) 42,000 units.
e) 50,000 units.

18. The equivalent units in the Assembly Department for direct materials for the
current month would be
a) 30,000 units.
b) 38,000 units.
c) 40,800 units.
d) 42,000 units.
e) 50,000 units.

19. The equivalent units in the Assembly Department for conversion costs for the
current month would be
a) 36,800 units.
b) 40,800 units.
c) 42,800 units.
d) 43,200 units.
e) 45,200 units.

Page 93 CMA Ontario – January 2008


Module 3 - Management Accounting

20. Butteco has the following cost components for 100,000 units of product for 20x0.

Raw material $200,000


Direct labor 100,000
Manufacturing overhead 200,000
Selling/administrative expense 150,000

All costs are variable except for $100,000 of manufacturing overhead and $100,000
of selling and administrative expenses. The total costs to produce and sell 110,000
units during 20x0 would be
a) $650,000.
b) $715,000.
c) $695,000.
d) $540,000.
e) $705,000.

21. The terms "direct cost" and "indirect cost" are commonly used in accounting. A
particular cost might be considered a direct cost of a manufacturing department but
an indirect cost of the product produced in the manufacturing department.
Classifying a cost as either direct or indirect depends upon
a) the behavior of the cost in response to volume changes.
b) whether the cost is expensed in the period in which it is incurred.
c) the cost objective to which the cost is being related.
d) whether an expenditure is unavoidable because it cannot be changed
regardless of any action taken.
e) the timing of the cash outlay for the cost.

22. In computing the current period's manufacturing cost per equivalent unit, the FIFO
method of process costing considers current period costs
a) Only
b) Plus cost of beginning work in process inventory
c) Less cost of beginning work in process inventory
d) Plus cost of ending work in process inventory

Page 94 CMA Ontario – January 2008


Module 3 - Management Accounting

Week 15
Multiple Choice Questions

1. The cost of defective units that normally arise under efficient operating conditions
should be
a) charged to administrative costs
b) prorated to cost of goods sold and ending inventory based on total actual units
produced
c) written off against owners' equity
d) prorated to cost of goods sold and ending inventory based on good units
produced
e) none of the above

2. Which of the following represents a major difference between activity-based


costing (ABC) and traditional product-based costing?
a) ABC accumulates indirect costs in many cost pools whereas traditional
product-based costing accumulates indirect costs in one or a few cost pools.
b) ABC assigns overhead costs based on cost drivers whereas traditional
product-based costing assigns overhead costs using cost application bases that
are not cost drivers.
c) ABC assigns overhead costs based on both financial and non-financial cost
drivers whereas traditional product-based costing assigns overhead costs
based on financial cost drivers only
d) ABC uses indirect application bases as cost drivers whereas traditional
product-based costing uses direct cost application bases as cost drivers.
e) None of the above

3. Activity-based costing in service organizations should be considered if


a) operating personnel have little faith in the accuracy of the existing cost
information.
b) they have a widely diverse set of operating activities.
c) services offered change frequently over time.
d) all of the above are true
e) only a and c above are true.

Page 95 CMA Ontario – January 2008


Module 3 - Management Accounting

The following information pertains to questions 4-5:

Ex Company, which produces a single product, began operations on January 1, Year 1.


Material A is added at the start of the production process and packaging material B is
added at the end of the process. Conversion costs are incurred uniformly throughout the
process. Inspection takes place when manufacturing is completed, but before packaging
material B is added. Spoiled units are discarded. Normal spoilage for this production
process is 4% of good output. Production data for the first quarter of Year 1 was as
follows:

Units started 18,000 units


Good units completed and transferred-out 15,000 units
Ending work-in-process inventory 2,000 units

Using a first-in, first-out (FIFO) process costing system, Ex Company incurred the
following costs per equivalent unit during the first quarter:

Material A $11.00
Material B $0.80
Conversion costs $15.00

The cost of ending work-in-process inventory using FIFO process costing was $34,000.

4. The loss from abnormal spoilage for the first quarter was
a) $16,080.
b) $10,720.
c) $10,400.
d) $15,600.
e) $26,800.

5. In terms of conversion, what was the percentage of completion of the ending work-
in-process inventory?
a) 65.4% complete
b) 34.7% complete
c) 54.5% complete
d) 40.0% complete
e) 63.4% complete

Page 96 CMA Ontario – January 2008


Module 3 - Management Accounting

6. Mesa Verde Co. had the following production for the month of June:

Work in process at June 1 20,000 units


Units Started during June 80,000
Completed and transferred to finished goods 66,000
Normal spoilage incurred 4,000
Work in process at June 30 30,000

Materials are added at the beginning of the process. As to conversion costs, the
beginning work in process was 70 percent complete and the ending work in process
was 60 percent complete. Spoilage is detected at the end of the process.

Using the weighted-average method, what were the equivalent units for June with
respect to conversion costs?
a) 84,000
b) 88,000
c) 90,000
d) 100,000

Page 97 CMA Ontario – January 2008


Module 3 - Management Accounting

The following information is for questions 7-8:

Scissors Inc. is a manufacturer of scissors. The company has always used a plant-
wide rate for allocating manufacturing overhead to its products. The plant manager
believes it is time to change to a better method of cost allocation. The accounting
department has been able to establish some new relationships between production
activities and the manufacturing overhead. They are as follows:

Activity Cost Driver Allocation Rate


Material handling Number of parts $ 4 per part
Assembly Labour hours $40 per hour
Inspection Time item is at $ 6 per minute
inspection station

The previous allocation method is based upon direct manufacturing labour hours,
and if that method is used the rate is $400 per labour hour.

7. What are the indirect manufacturing costs per scissors assuming the traditional
method is used and a batch of 1,000 scissors were produced? The batch requires
2,000 parts, 20 direct manufacturing labour hours, and 30 minutes of inspection
time.
a) $ 8.00
b) $ 9.80
c) $8,000.00
d) $8,980.00
e) none of the above

8. What are the indirect unit manufacturing costs of a batch of 100 scissors assuming
the activity base method is used? The batch requires 200 parts, 12 direct
manufacturing labour hours, and 5 minutes of inspection time.
a) $ 8.00
b) $ 13.10
c) $ 48.00
d) $1,310.00
e) none of the above

Page 98 CMA Ontario – January 2008


Module 3 - Management Accounting

Week 16
Multiple Choice Questions

1. B.F. Spurs Co. has two service departments, A and B, and two production
departments. The two service departments render services to each other as well as
to the production departments. The costs of service department A are allocated to
the other three departments based on their direct usage of department A's services.
The costs of service department B are then allocated to the production departments.
Which of the following cost allocation methods is being used to allocate the service
departments' costs?
a) Dual-rate
b) Direct
c) Step-down (sequential)
d) Activity-based
e) Reciprocal (simultaneous-equations)

The following information pertains to questions 2-3:

Omega Company manufactures three chemicals in a joint process. The


manufacturing costs of the joint process include $25,000 of direct materials and
$35,000 of conversion costs. All three chemicals are then processed further before
they are sold. Other pertinent data are as follows:

Chemicals Sales Value at Split-off Separable Costs Final Sales Value


A $50,000 $28,000 $100,000
B 40,000 10,000 60,000
C 30,000 12,000 40,000

2. Using the estimated net realizable value method, the joint costs allocated to
Chemical A would be
a) $16,800
b) $25,000
c) $28,800
d) $30,000
e) $33,600

3. The decision to process all three chemicals beyond the split-off point is suboptimal.
If the optimal decision had been made, the income of Omega Company would have
improved by
a) $2,000
b) $10,000
c) $30,000
d) $60,000
e) an amount different from the above amounts

Page 99 CMA Ontario – January 2008


Module 3 - Management Accounting

4. The main purpose of allocating joint costs to products is to


a) determine prices for the products
b) make decisions about dropping the products
c) rationalize economic decisions concerning the products
d) determine the cost of the products
e) determine budgets for the production of the products

5. A business uses the step-down method to allocate service department costs to the
manufacturing departments. Assume there are two service departments and two
manufacturing departments, as shown below:

Service Manufacturing
Departments Departments
Plant Custodial
Admin. Services Cutting Polishing
Costs $360,000 $90,000 $261,000 $689,000
Labour hours 25,000 6,000 18,000 30,000
Space occupied (m2) 10,000 1,000 5,000 45,000

Plant administration costs are allocated based on labour hours, and custodial
services costs are allocated based on space occupied. The total costs of the cutting
and polishing departments (rounded to the closest .000), after allocating all the
service department costs, starting with the largest service provider are

a) cutting - $396,000, and polishing - $914,000.


b) cutting - $405,000, and polishing - $995,000.
c) cutting - $381,000, and polishing - $889,000.
d) cutting - $394,000, and polishing - $1,006,000.
e) cutting - $380,000, and polishing - $892,000.

Page 100 CMA Ontario – January 2008


Module 3 - Management Accounting

The following information pertains to questions 6-7:

SMT Ltd. manufactures three products. Production begins with a joint process and the
three outputs of the joint process are processed further to produce products L, M and N.
The outputs at split-off have no market value. Last year, the joint costs amounted to
$600,000. Other data for last year are as follows:

Product L Product M Product N


Selling price per unit $160 $300 $400
Costs per unit after split-off to complete and sell $100 $200 $350
Total output at split-off used in production 16,400 kg 10,000 kg 8,400 kg
Production in units 20,000 10,000 7,000
Sales in units 18,000 8,000 7,000

6. Using the physical quantities method (also called the physical measure method or
the average method) of joint costing, the total joint costs allocated to the production
of product L last year was

a) $200,000.
b) $327,273.
c) $282,759.
d) $324,324.
e) $282,353.

7. Using the estimated (approximate) net realizable value method of joint costing, the
inventory cost per unit of product M is

a) $226.91.
b) $223.53.
c) $221.52.
d) $220.00.
e) $217.24.

Page 101 CMA Ontario – January 2008


Module 3 - Management Accounting

The Following Data Apply to Items 8-9:

The managers of Rochester Manufacturing are discussing ways to allocate the cost of
service departments such as Quality Control and Maintenance to the production
departments. To aid them in this discussion, the controller has provided the following
information.
Quality
Control Maintenance Machining Assembly Total

Budgeted overhead costs


before allocation $350,000 $200,000 $400,000 $300,000 $1,250,000

Budgeted machine hours - - 50,000 - 50,000

Budgeted direct labor - - - 25,000 25,000


hours

Budgeted hours of service:

Quality Control - 7,000 21,000 7,000 35,000


Maintenance 10,000 - 18,000 12,000 40,000

8. Using the direct method, the total amount of overhead allocated to each machine
hour in the Machining Department would be
a) $2.40
b) $5.25
c) $8.00
d) $9.35
e) $15.65

9. If Rochester Manufacturing uses the step-down method of allocating service costs


beginning with Quality Control, the Maintenance costs allocated to the Assembly
Department would be
a) $70,000
b) $108,000
c) $162,000
d) $200,000
e) $210,000

Page 102 CMA Ontario – January 2008


Module 3 - Management Accounting

10. Raynor Manufacturing purchases rough-cut trees and processes them up to the
splitoff point, where two products (paper and pencil casings) are obtained. The
products are then sold to an independent company that markets and distributes them
to retail outlets. In October, Raynor processed 50 trees (yield is 30,000 sheets of
paper and 30,000 pencil casings and no scrap), and had the following production
and sales:

Production: 30,000 sheets of paper and 30,000 pencil casings


Sales: 29,000 sheets of paper at $0.04 per page
30,000 pencil casings at $0.10 per casing

Cost of purchasing 50 trees and processing them up to the splitoff point to yield
30,000 sheets of paper and 30,000 pencil casings is $1,500. Opening inventories are
nil.

What are the approximate joint costs assigned to the paper ending inventory if joint
costs are allocated using the sales value at splitoff method?
a) $14.29
b) $50.00
c) $435.00
d) $750.00
e) $915.00

Page 103 CMA Ontario – January 2008


Module 3 - Management Accounting

11. Beverage Drink Company processes direct materials up to the splitoff point, where
two products, A and B, are obtained. The following information was collected for
the month of July:

Direct materials processed: 2,500 litres (with 20 percent shrinkage)

Production: A 1,500 litres


B 500 litres
Sales: A $15.00 per litre
B $10.00 per litre

Cost of purchasing 2,500 litres of direct materials and processing it up to the


splitoff point to yield a total of 2,000 litres of good products was $4,500. There
were no inventory balances of A and B.

Product A may be processed further to yield 1,375 litres of Product Z5 for an


additional processing cost of $150. Product Z5 is sold for $25.00 per litre. There
was no beginning inventory and ending inventory was 125 litres.

Product B may be processed further to yield 375 litres of Product W3 for an


additional processing cost of $275. Product W3 is sold for $30.00 per litre. There
was no beginning inventory and ending inventory was 25 litres.

What is Product Z5's estimated net realizable value at the splitoff

a) $11,100
b) $22,350
c) $24,225
d) $29,375
e) $34,225

Page 104 CMA Ontario – January 2008


Module 3 - Management Accounting

Introduction to Management Accounting

Management vs. Financial Accounting

• internal focus: provision of accounting


information to management
• level of detail is much higher
• no fixed set of rules
• leading indicators vs. lagging indicators of
performance
• includes non-financial information
• focus on decision making and control

What Management Accounting is about?

• cost systems - how much does it cost to provide a


certain service, to manufacture a given product?
• planning decisions - what tasks should be
performed and how to complete them
• control decisions - how to manage, motivate and
monitor individuals; encompasses the choice of
performance measures and reward system
– monitoring
» direct: observation of what people do
» indirect: observation of people’s output

© Jacques Maurice, 2008


105
Module 3 - Management Accounting

Conflict between planning and control

• managers have specialized information useful for


planning decisions
– i.e. to make the best planning decision, we
need managers to divulge their specialized
(private) information
• but any information divulged will be used to
evaluate the manager…. therein the conflict!

The Three-Legged Stool

Control systems within organizations:


1. Assigning responsibilities
2. Measuring performance, and

3. Providing compensation for performance


Control is the process of getting members of the
organization to work towards the goals of the
organization

The 3 Functions of Management Accounting

• problem solving: identification of the best


alternatives in relation to the organization’s goals
• scorekeeping: the accumulation of data and
reporting reliable results to all levels of
management
• attention directing: identification of situations
requiring management attention

© Jacques Maurice, 2008


106
Module 3 - Management Accounting

Cost Classification

Cost Classifications

• cost object: a product, contract, or organizational


segment for which costs are determined
• product vs. period costs
– product: manufacturing costs consisting of
direct labour, direct materials and overhead;
inventoriable
– period costs: non manufacturing costs -
selling, general and administrative costs - that
are expenses in the accounting period

Cost Classifications - cont’d

• direct vs. indirect costs


– direct: cost that can be obviously and
physically traced to a cost object
– indirect: cost that must be allocated in order to
be assigned to a cost object
• variable vs. fixed costs
– variable cost: varies in total in direct
proportion to changes in volume
– fixed costs: remains constant in total
regardless of the volume of activity
9

© Jacques Maurice, 2008


107
Module 3 - Management Accounting

Cost Classification Exercise

Classify each of the following as product/period,


direct/indirect, variable/fixed (cost object = car; 2 cars; 2
assembly lines)
1. Cost of tires
2. Insurance on manufacturing equipment
3. Cost of steel used to make body
4. Quality inspection
5. Wages of materials-handling workers
6. Depreciation on factory building
7. Salary of CEO

10

Cost Classification Exercise - cont’d

8. Wages of assembly line workers


9. Salaries of plant security personnel
10. Overtime premiums paid to assembly line workers
11. Cost of electric power to operate assembly line machinery
12. Glue used in production
13. Salary of engineers who design changes on cars
14. Idle time of assembly line workers
15. Wages paid to temporary assembly line workers
16. Annual fire insurance policy on plant

11

Cost Classifications - cont’d

• controllable vs. noncontrollable costs


– a cost is controllable at a particular level of
management if that level has power to authorize the
cost
• step costs - costs that increase by increments at fixed
intervals
– small steps (variable) vs. large steps (fixed)
• allocated cost: a cost assigned to several cost objects

12

© Jacques Maurice, 2008


108
Module 3 - Management Accounting

Cost Classifications - cont’d

• sunk cost: a cost that has already been incurred


and that cannot be changed by any decision
made now or in the future
• opportunity cost: the potential benefit that is lost
or sacrificed when the selection of one course of
action makes it necessary to give up a competing
course of action

• relevant range: the range of activity within which


assumptions relative to variable and fixed cost
behaviour is valid
13

Product Costs

• direct (raw) materials: those materials that


become an integral part of a finished product and
that can be conveniently traced into it

• direct labour: those factory labour costs that can


be physically traced to the creation of products in
a hands-on sense

• manufacturing overhead: all other


manufacturing
• prime costs = direct labour + direct materials
• conversion costs = direct labour + overhead 14

Types of Inventories

• manufacturing company:
– direct materials inventory
– work-in-process inventory
– finished goods inventory
• merchandising company:
– finished goods inventory

15

© Jacques Maurice, 2008


109
Module 3 - Management Accounting

Cost Flows

Direct Materials Direct Labour Manufacturing Overhead

Work in process

Finished Goods

Cost of Goods Sold


16

Statement of Cost of Goods


Manufactured

Direct Materials Used


Direct Materials, Opening Inventory XXX
Purchases of Direct Materials XXX
Direct Materials, Ending Inventory -XXX
XXX
Direct labour XXX
Manufacturing overhead XXX
Total Manufacturing Costs XXX
Work-in-Process, beginning XXX
Work-in-Process, ending -XXX
Cost of goods manufactured XXX

17

Types of Costing Systems

Actual Normal Standard


Costing Costing Costing

Direct Actual Actual Budgeted


Costs

Indirect Actual Budgeted Budgeted


Costs

18

© Jacques Maurice, 2008


110
Module 3 - Management Accounting

Job Order Costing

19

Job Order Costing System

• job order costing is used by companies making


products according to user specifications
• costs are accumulated individually by job:
– costs of different jobs are maintained in
separate subsidiary ledger accounts
– direct materials, direct labor, and overhead
are accumulated for each job.
– the normal costing method of valuation is
used since actual direct material and direct
labor costs are fairly easy to identify and are
associated with a particular job
20

Job Order Costing - Details and


Documents

• the job order cost sheet is a source document that


provides virtually all the financial information
about a particular job; the set of all job order cost
sheets for uncompleted jobs composes the Work
in Process subsidiary ledger
• direct materials information is gathered from the
materials requisition forms, while direct labor
information is found on employee time sheets or
employee labor tickets.

21

© Jacques Maurice, 2008


111
Module 3 - Management Accounting

Job Order Costing - Details and


Documents (cont’d)

• overhead is applied to production using


predetermined overhead rates
• completion of production: job cost sheets for
completed jobs are removed from the Work in
Process subsidiary ledger and are transferred to
the Finished Goods file and serve as a subsidiary
ledger for that account.

22

The Predetermined Overhead Rate

• POR = Budgeted Overhead


÷ Expected Input Volume
• the input measure selected has to:
– be common to all products manufactured and,
– should have the greatest association or
causality with overhead
• typical input measures: direct labour hours or
dollars, machine hours, material cost

23

Overhead Application

• overhead is applied using the predetermined rate


times the actual level of activity

Manufacturing Overhead

Actual Overhead
Overhead Applied =
Costs Actual Input
Incurred Volume x POR

Debit Balance = Credit Balance =


UnderApplied OverApplied
Overhead Overhead
24

© Jacques Maurice, 2008


112
Module 3 - Management Accounting

Overhead Application - cont’d


• the amount of under/over applied overhead is usually
disposed of at year end
– if the amount is not material
» it can be closed to COGS, or
» it can be carried forward to next year
– if material, it must allocated to the accounts containing
applied overhead: WIP, finished goods inventory and
COGS, 2 approaches -
» allocate to the balances based on overhead
allocated (if known), or
» prorate to balances - net of opening WIP and
opening finished goods inventory

25

Process Costing

26

Process Costing

• information provided is highly aggregated and


not too useful for decision making
• equivalent unit: the amount of output stated in
terms of completed units

27

© Jacques Maurice, 2008


113
Module 3 - Management Accounting

Process Costing Example

• assembly department incurs only labour costs


• costs added during period = $14,500
• costs in opening WIP = $1,536
• unit flow:
– units in opening WIP = 400 (40% complete)
– units started = 1,500
– units in ending WIP = 250 (80% complete)

28

Process Costing - Weighted Average

• assumes that the opening WIP is 0% complete

• unit cost = (costs in beginning inventory + costs


added) ÷ equivalent units

• equivalent units =
units transferred out
+ EU in ending inventory

29

Process Costing - Weighted Average (cont’d)

• cost of units transferred out =


– units transferred out x total unit cost

• cost of ending inventory =


– equivalent units in ending inventory, by
category, x unit cost for that category

30

© Jacques Maurice, 2008


114
Module 3 - Management Accounting

Process Costing - FIFO

• unit cost = costs added ÷ equivalent units

• equivalent units =
work done to complete opening inventory
+ Units started and completed
+ EU in ending inventory

31

Process Costing - FIFO (cont’d)

• cost of units transferred out =


– costs in opening WIP +
– cost to complete opening WIP: equivalent
units to complete, by category, x unit cost for
that category +
– units started and completed X total unit cost
• cost of ending inventory =
– equivalent units in ending inventory, by
category, x unit cost for that category

32

Spoilage

33

© Jacques Maurice, 2008


115
Module 3 - Management Accounting

Spoilage Terminology

• spoilage - unacceptable units of production that


are discarded or sold at reduced prices
• reworked units - unacceptable units of
production that are subsequently repaired and
sold as acceptable finished goods
• scrap - material left over when making a product
that has low sales value

34

Normal vs. Abnormal Spoilage

• normal spoilage - spoilage as an inherent result


of a particular production even under efficient
operating conditions -> expected spoilage
– treat as a component of unit cost of good units
manufactured
• abnormal spoilage - spoilage that should not
arise under efficient operating conditions ->
avoidable and controllable
– do not treat as a product cost

35

Accounting for Spoilage


Job Costing

• normal spoilage attributable to a specific job - job


bears the cost of the spoilage reduced by the
current net realizable value of the spoilage
• normal spoilage common to all jobs - treat as
manufacturing overhead
• abnormal spoilage - charge to an abnormal loss
account

36

© Jacques Maurice, 2008


116
Module 3 - Management Accounting

Cost Estimation

37

Steps in Estimating a Cost Function

• choose the dependent variable -y


– ideally, all the individual items in the
dependent variable will have a similar
relationship with the cost driver(s), chosen in
the next step
– example: cost driver to predict the total cost of
health benefits could be the number of
employees

38

Steps in Estimating a Cost Function -


cont’d

• choose the cost driver(s) - x , x , …, x


1 2 n

– the chosen driver should be economically


plausible and accurately measurable
– there should be a cause and effect relationship
between the cost driver and the resulting costs

39

© Jacques Maurice, 2008


117
Module 3 - Management Accounting

Steps in Estimating a Cost Function -


cont’d

• collect data on the dependent variable and on the


cost driver(s) - y, x i

– the ideal database would contain numerous


observations in periods not subject to major
technological change
– the time period used to measure the
dependent variable and the cost driver(s)
should be identical

40

Steps in Estimating a Cost Function -


cont’d

• plot the data - a plot will reveal whether the cost relation is
indeed linear, and whether there are any outliers
• high-low method: makes use of the costs and activity levels
for the high and low activity levels in a set of data
– variable cost = ∆ in cost / ∆ in activity level between the
highest and lowest points of activity
– relies only on two data points
• visual fit (scattergraph) method: visually fit a straight line
in a scatter plot of data
• regression analysis: most reliable, as it uses all data points
in calculating the best fitting line

41

Cost Estimation - Example


Month MH Material $ Overhead $
1 175 $4,750 $4,500
2 170 4,600 4,225
3 160 4,200 3,780
4 190 5,900 5,250
5 175 4,600 4,800
6 200 5,250 5,100
7 160 4,350 4,450
8 150 4,350 4,200
9 210 6,000 5,475
10 180 4,950 4,760
11 170 4,450 4,325
12 145 3,800 3,975 42

© Jacques Maurice, 2008


118
Module 3 - Management Accounting

43

44

45

© Jacques Maurice, 2008


119
Module 3 - Management Accounting

46

Cost Driver = Machine Hours

47

Cost Driver = Material $

48

© Jacques Maurice, 2008


120
Module 3 - Management Accounting

Cost Driver = MH + Material $

49

Regression Analysis

• criterion 1: Economic Plausibility


– the basis relationship between the dependent
variable and the independent variable should
make economic sense and be intuitive to both
the operating manager and the management
accountant

50

Regression Analysis - cont’d

• criterion 2: Goodness of Fit


– does the independent variable explain a
substantial percentage of the variation in the
independent variable?
– the coefficient of determination, r is used to
2

measure the goodness of fit


– the r test is a measure of the extent to which
2

the independent variable explains or accounts


for the variability of the of the dependent
variable

51

© Jacques Maurice, 2008


121
Module 3 - Management Accounting

Regression Analysis - cont’d

• criterion 3: Significance of Independent


Variable(s)
– the coefficient of the chosen independent
variable(s) should be significantly different
from zero, implying an important relationship
exists between independent and dependent
variables - measured by the t-value of the
coefficient of the independent variable(s)

52

Activity Based Costing

53

Activity Based Costing


• why traditional cost systems fail?
– overhead is typically allocated to products using unit-
based measures such as direct labor, materials
purchases, processing time or units produced
– the product costs produced by such allocations are
distorted because products do not consume most
support resources in proportion to their production
volumes
• distortions are most severe in organizations producing a
diverse product mix
• products that differ in volume, complexity, and age
consume support resources in significantly different
amounts

54

© Jacques Maurice, 2008


122
Module 3 - Management Accounting

Activity Based Costing - Structural


Overview
COSTING Resource
SYSTEM Categories

Resource
Drivers

Activity Perf Eval


Activities
Analysis for ABM

Activity COST
Drivers MANAGEMENT

Direct Cost 55
Costs Objects

The Structure of ABCS

• first stage assigns the expenses of support resources to the


activities performed by these resources

– assumption is that activities causes costs

• in the second stage, activity costs are assigned to products


based on individual products' consumption or demand for
each activity

– assumption is that products create the demand for


activities

56

Classes of costs: Unit and Batch


• unit cost: costs that vary proportionally with the number of
units produced
– examples: direct material, direct labor, machine hours
– these costs are proportional to short-run production
decisions
• batch related costs are incurred each time a batch of goods
is processed
– examples: setups, material movements, purchase
orders, inspection
– the demand for setup resources are independent of the
number of units produced after completing the setup

57

© Jacques Maurice, 2008


123
Module 3 - Management Accounting

Classes of costs: Product and Facility

• product related costs: incurred to enable individual


products to be produced and sold
– the expenses of these activities can be traced to the
individual products, but the resources consumed by the
activities are independent of how many units or batches
of the products are produced
– examples: process engineering, product specifications,
product enhancements
• facility related costs: costs incurred to create productive
capacity for all products
– examples: factory administration, maintenance, heating
and lighting

58

When to Implement Activity Based


Costing

• ABCS are likely to report significantly more accurate


product costs when...
– the organization uses large amounts of indirect
resources in its production processes, and/or
– the organization has significant diversity in products,
production processes, and customers, and/or
– the company faces significant competition, especially
from more focused competitors

59

Benefits of Activity Based Costing

• improved decisions: w.r.t introduction, pricing and


discontinuance of products; ABCS eliminates cross-product
subsidies
• continuous improvement activities to reduce overhead
costs: managers will now know what activities causes costs
and can reduce or eliminate those activities that add little
or no value to products or services
• ease of determining relevant costs: product cost data in
traditional systems often needs to be analyzed in order to
obtain information relevant to a special decision; under
ABCS, the data is readily available

60

© Jacques Maurice, 2008


124
Module 3 - Management Accounting

Cost Allocations

61

Cost Allocations
• cost allocation: the assignment of indirect, common or joint costs
(costs that cannot be unequivocably attributed to a final cost object) to
a cost object (department, process of product/service)
• used as a proxy to hard-to-observe opportunity costs
• common costs: costs of a facility that provides services for two or
more cost objects
• cost attribution: process of assigning a direct or attributable cost to a
cost object
• ALL COST ALLOCATIONS ARE ARBITRARY

62

Allocating Service Department Costs

• service department organizational unit that provide most,


or all of the services to customers inside the organization
• operating departments: organizational units that provide
most, or all of their services to customers outside the
organization
• the cost of a good or service provided to an outside
customer includes the cost of the direct services provided
by the operating departments and the cost of the indirect
services provided by the service departments

63

© Jacques Maurice, 2008


125
Module 3 - Management Accounting

Allocating Service Department Costs -


cont'd

• service departments should allocate their costs to the


departments that use their services in proportion to the
causal factor explaining the costs being allocated
• variable costs should be allocated in proportion to actual
use of the department's services, which is the causal factor
that explains the behavior of variable costs
• fixed costs should be allocated in proportion to planned
use of the department's services, which is the causal factor
that explains the behavior of fixed costs

64

Allocating Service Department Costs -


cont'd

• each service department should maintain


separate cost pools for its fixed costs and its
variable costs
• a costing system should allocate each service
department's fixed and variable costs separately
• three approaches used to allocate service
department costs: direct method, step method
and the reciprocal method

65

66

© Jacques Maurice, 2008


126
Module 3 - Management Accounting

Direct Method
• allocates costs directly from the service department to the
operating departments ignoring services provided to itself and
to other service departments

• methodology: allocate the service department’s costs to the


operating departments based on the operating department’s
use of the service department

• problems: (1) the opportunity cost per unit of service charged is


wrong, (2) service departments will overuse the other service
departments

67

Step Method

• allocates costs in steps from the service department to the


operating departments - gives partial consideration to the
services provided to other service departments
• methodology:
– rank the service departments (arbitrary)
– determine each service department's costs (equal to the
service department's initial costs + any costs allocated
from other service departments)

68

Step Method - cont’d

• methodology (cont’d) -
– determine the total units of service provided by the
chosen department to the other departments, both
operating and service; allocate; then delete that dept
from the analysis
– all service department costs eventually cascade through
the service organizations and are eventually allocated
to the operating departments

69

© Jacques Maurice, 2008


127
Module 3 - Management Accounting

Step Method - cont’d

• problems:
– sequence used is arbitrary
– once a department is allocated it does not receive a
subsequent allocation
– conflict of interests among operating department
managers
– incentive effects: the cost per service unit may bear little
or no relationship with the true opportunity cost

70

Reciprocal Method

• most precise method and captures all service flows


between service departments
• a system of linear equations is constructed, one for
each service department - each equation contains
the use of that department by all other service
departments
• by incorporating one service department’s use of
another service department, it avoids the
distortions in unit allocated cost observed in the
step-down method

71

Reciprocal Method - cont’d

• produces a transfer price that can be compared


with an outside price for the service (assuming
there are no unavoidable fixed costs)
• although theoretically better, historically not
used due to:
– access to computing power,
– accountants not trained, or
– difficult to explain to nonfinancial managers

72

© Jacques Maurice, 2008


128
Module 3 - Management Accounting

Joint and By-Product Costing

73

Joint Costs

• joint costs - costs of a single production process


that yields multiple products
• splitoff point - juncture in the process when
products become separately identifiable
• separable costs - all costs incurred beyond the
point that are assignable to one or more
individual products

74

Why Allocate Joint Costs?

• financial accounting and tax purposes: inventory


costing and COGS computations
• inventory costing and COGS computations used
internally
• cost reimbursement
• insurance settlement computations, rate
regulation, etc…
• but NOT required for decision making purposes

75

© Jacques Maurice, 2008


129
Module 3 - Management Accounting

Allocation Methods

• market based:
– net realizable value (NRV)
– sales value at splitoff
– constant gross-margin % NRV
• measures of physical volume or mass
• economic decision making, however, does not
require cost allocations

76

Methodology

• net realizable value for each product = final sales product


of each product less any associated costs to process further;
calculated at the split off point
• sales value at splitoff - used when all products are sold at
the split-off point
• constant gross-margin % NRV:
– calculate the gross margin for the overall process
– deduct the gross margin % from the final sales value;
this results in the total cost allocated to each product
– deduct the separable costs to obtain the allocation of
joint costs

77

Joint-Cost Allocation Example

Chemical B
$85 / l

Chemical A Process Chemical C Process $100 / l


$20,000 $20 / l
$17 / l
Chemical D
$120 / l

Each batch of production consists of 1,000 litres of Chemical A and produces


200 litres of Chemical B, 500 litres of Chemical C and 300 litres of Chemical D.

78

© Jacques Maurice, 2008


130
Module 3 - Management Accounting

Which Method to use?

• if joint costs are sold at the split-off point, use the


sales value at splitoff method
• otherwise, the estimated NRV or constant gross-
margin % should be used
• physical measures of volume or mass can lead to
significant distortions

79

ByProducts

• byproduct - a product which has relatively low


sales value compared to a joint product (i.e.
sawdust)
• method A - reduce joint costs by the NRV of the
byproducts; byproducts get recorded in
inventory at their NRV
• method B - do not record byproducts in
inventory; recognize revenues when sold

80

© Jacques Maurice, 2008


131

You might also like